You are on page 1of 231

COURSE PACK

FOR
FOUNDATION OF BUSINESS MATHEMATICS

COURSE CODE: 105


COURSE: BBA
SEMESTER 1
YEAR: 2023-2024

COURSE INSTRUCTORS: Mr. Amit Grover


Ms. Taruna Gulati
Ms. Veenu Gupta
Mr. Mayur Taneja

Course Leader: Mr. Mayur Taneja

Forwarded by: - Forwarded by: - HOD Approved by: Director

Dr. Aarushi Kataria Dr. Parul Aggarwal Prof. (Dr.) Yamini Agarwal
Programme Coordinator

Bharati Vidyapeeth (Deemed to be University) Institute of Management and


Research, New Delhi
An ISO 9001:2015 &14001:2015 Certified Institute
A-4, Paschim Vihar, ND-110063 (Ph. 01125284396, 25285808)

“Strictly For Internal Academic Use”


FACULTY PROFILE:

Mr. Amit Grover is a dedicated and experienced educational professional, currently


working as a visiting faculty at Bharati Vidyapeeth Deemed University, New Delhi since
2008. A post graduate in commerce, with 20+ years of experience in teaching young
adults across a wide range of age groups and diverse backgrounds. Adept at creating
powerful curriculum in the field of Finance Statistics and Mathematics. Knowledgeable
and experienced in various educational philosophies, which best promote the overall
experience of a student. A committed faculty member, passionate about working to
further enhance the educational offerings of an institution.

Ms. Taruna Gulati is B.Sc., B.Ed., and M.Sc. (Mathematics) degree holder. Moreover,
she is pursuing Ph.D. (Mathematics) from Amity University. She is working as PGT
(Maths) in a reputed school in Delhi. She is a visiting faculty in Bharati Vidyapeeth
University, New Delhi. She has more than 14 years of teaching experience. She has
published many papers in reputed National and International journals. She has a
passion of teaching.

Mr. Mayur Taneja holds a M. Com and B. Com (Hons.) degree from Delhi University. He
is working as an Assistant Professor in a reputed college of University of Delhi. He has
more than ten years of teaching and industry experience. He has been working as a
visiting faculty in Bharati Vidyapeeth University, New Delhi since last 5 years. He has
published many research papers in National and International journals and has co-
authored a chapter in a book.

Ms. Veenu Gupta is MBA, MA (ECONOMICS), BA (ECO. HONS) degree holder. She is also
NET qualified in Management as well as in Economics. Now she is pursuing PhD from
State University, BPSMV, Sonepat. She is working as an Assistant Professor in Bharati
Vidyapeeth University. She has more than 7 years of teaching experience. Her area of
interest is Economics, Business Mathematics, Business Statistics, etc.
INDEX

S.No Topic Page No.

1. Foundation of Business 1-2


Mathematics Syllabus
2. Course Outline 3-7
3. Study Material Unit 1 8-44

4. Study Material Unit 2 45-88


5. Study Material Unit 3 89-139
6. Study Material Unit 4 140-191
7. Study Material Unit 5 192-220
8. Previous year papers (Internals and 221-227
University)
1
2
Bharati Vidyapeeth Institute of Management and Research, New
Delhi

BBA - 1st semester, Academic Year 2023-24

1. Course Code: 105

2. Course Title: Foundation of Business Mathematics

3. Course Overview:

Business Mathematics is mathematics used by commercial enterprises to record add


manage business operations. Commercial organizations used mathematics in
accounting, inventory management, marketing, sales forecasting, and financial
analysis. Mathematics typically used in commerce includes elementary automatic,
element elementary, algebra and probability. The objectives of course are to give a
fundamental knowledge of the basic concepts of mathematics which can be applied
or described in real life phenomena.

It aims to equip the students with the mathematical background for business
management and to know the rules of mathematics in areas of business-like
accountancy, economics, management science, etc.

3
4. Learning Outcomes:
1. To develop Knowledge of key theories, concepts in Mathematics.
2. To enhance ability to problem solving.
3. To build ability to apply mathematical concept for business application.

5. List of Topics / Modules

Topic / Module Contents / Concepts


Module 1: Commercial Arithmetic Ratio: Definition, meaning. Working
examples Proportion: Definition, Types of
proportion, Working examples Percentage:
Meaning, Working examples Partnership:
Meaning, Working examples
Module 2: B u s i n e s s Profit: Meaning, Working examples Loss:
Mathematics Meaning, Working examples Discount:
Meaning, Types of Discount, Working
examples Commission: Meaning, Types of
Commission agents, Working examples
Brokerage: Meaning, Working examples
Payroll: Meaning, Working examples
Module 3: Financial Mathematics Simple Interest: Meaning, working
examples Compound Interest: Meaning,
working examples on Interest Compounded
Continuously, Compound Amount at
changing rate

Module 4: Matrices and Determinants Matrix: Definition of a Matrix, Matrix


&Simultaneous Linear equations operations, Working examples
Determinants: Definition, Properties of
determinants. Applications in Business
Problem, Solution of Simultaneous
equations, Working examples
Module 5: Time Value of Money and Introduction of Annuity, Types of Annuity,
Simple Annuity Amount and Present Value of Immediate
(NPV), Annuity, Annuity Due

4
6. Evaluation criteria
Component Description Weightage
1. End Term Exams It will be based on 60%
conceptual questions,
situation specific
application-oriented
questions and short case
studies, End term exams
cover both pre-mid-term
and post mid-term course
coverage. Course readings
are an integral
component of learning in
this course.
2. Internal Exam I It will be based on 10%
conceptual questions,
situation specific
application
orientedd questions and
short case studies. Course
readings are an integral
componentof learning in
this course. At least one of
the questions will be
based onthese readings
which will not be
specified to the students.
This shall cover half of the
Syllabus.
3. Internal Exam II This would cover other 10%
half of the syllabus.

4. Attendance & Class Students will be awarded 10%


participation marks on the basis
of 75% attendance
through ERP.

5. CES Class test will be 10%


conducted as part of CES.
One of these shall be
administered before first
internal, and two post
5
first internals. Average
marksof these three class
testswill be considered
for course grades.

6. Recommended / Reference Text Books and Resources:

Text Books Business Mathematics, Agrawal

Reference Books ⚫ Business Mathematics, Dr. Amarnath


Dikshit & Dr. Jitendra Kumar Jain
⚫ Business Mathematics, Nirmala M,
Gurunath Rao Vaidya, and Nirmala
Joseph (2021)
⚫ Business Mathematics and Statistics,
Dr.Sancheti& Kapoor
⚫ Business Mathematics, Agrawal
⚫ Business Mathematics, Azharuddin

6
7. Contact Details:

Name of the Instructor Mr. Amit Grover, Ms. Taruna Gulati,


Mr. Mayur Taneja, Ms. Veenu Gupta

Office Location Visiting Faculty

Telephone 9810377604, 9999194210, 9971806488,


9873511578

Email amit7604@gmail.com,
tarunagulati07@gmail.com,
Mayurtaneja88@gmail.com,
Goel.veena4@gmail.com

7
UNIT 1:
COMMERCIAL ARITHMETIC
(Ratio: Definition, meaning.
Working examples
Proportion: Definition, Types of
proportion, working examples
Percentage: Meaning, Working
examples
Partnership: Meaning, Working
Examples)

8
RATIO AND PROPORTION
A ratio is the comparison or simplified form of two quantities of the same
kind. This relation indicates how many times one quantity is equal to the
other; or inother words, ratio is a number, which expresses one quantity as
a fraction of the other. E.g. Ratio of 3 to 4 is 3 : 4. The numbers forming
the ratio are called terms. The numerator, “3”, in this case, is known as the
antecedent and the denominator, “4”, in this case, is known as the
consequent.

• Equivalent Ratios Let us divide a Pizza into 8 equal parts and share
itbetween Ram and Sam in the ratio 2:6. The ratio 2:6 can be written
as 2/6;2/6 = 1/3 We know that 2/6 and 1/3 are called equivalent
fractions. Similarly we call the ratios 2:6 and 1:3 as equivalent
ratios.
From a given ratio x : y, we can get equivalent ratios by multiplying
theterms ‘x’ and ‘ y ‘by the same non-zero number.
For example

1:3=2:6=3:9
4 : 5 = 12 : 15 = 16 : 20

Example 1: Write any 4 equivalent ratios for 4 : 3.


Sol: Given Ratio = 4 : 3. The ratio in fractional form = 4/3, we can get
equivalent ratios by “4”and “3” by 2, 3, 4, 5 and get the equivalent
fractions of4/3 are 8/6, 12/9, 16/12, 20/15,
∴ The equivalent ratios of 4 : 3 are 8 : 6, 12 : 9, 16 : 12, 20 : 15
Example 2: Distribute Rs. 320 in the ratio 1 : 3.
Sol: 1 : 3 means the first quantity is 1 part and the second quantity in 3
parts.The total number of parts = 1 + 3 = 4. As 4 parts = Rs. 320
∴ 1 part = 320/4 = 80 ∴ 3 parts = 3 × 80 = Rs. 240

• If a : b is a ratio then:
✔ Duplicate ratio of (a : b) is (a^2 : b^2).
✔ Sub-duplicate ratio of (a : b) is (a^1/2 : b^1/2).
✔ Triplicate ratio of (a : b) is (a^3 : b^3).
✔ Sub-triplicate ratio of (a : b) is (a^1/3 : b^1/3).

Example 3: What is the duplicate ratio of 2 : 3?


Sol: Duplicate ratio of 2 : 3 = 22 : 32 = 4 : 9.
Example 4: Triplicate ratio of two numbers is 27 : 64. Find their duplicate
ratio.Sol: Triplicate ratio of two numbers is 27 : 64, so numbers should be
271/3 : 641/3 So numbers are in the ratio 3 : 4. So duplicate ratio of 3 : 4 = 32
9
: 42 = 9 : 16.
Example 5: The ratio of two numbers is 25 : 36. Find their sub duplicate ratio.

Sol: Sub duplicate ratio of 25 : 36 = 251/2 : 361/2 = 5 : 6.

10
• PROPORTION
Proportion is represented by the symbol ‘= ‘or ‘:: ‘
If the ratio a : b is equal to the ratio c : d, then a, b, c, d are said to be
inproportion.
Using symbols we write as a : b = c : d or a : b :: c : d
• When 4 terms in proportion, then the product of the two extremes
(i.e.the first and the fourth value) should be equal to the product of
two middle values (i.e. the second and the third value)

Example 6: Prove that 16 : 12 and 4 : 3 are in proportion.


Sol: The product of the means = 12 × 4 = 48. The product of the extremes
=16 × 3 = 48
As Product of Means = Product of Extremes ∴ 16 : 12, 4 : 3 are in proportion.
Example 7: Find the missing number in 3 : 4 = 12 :
Sol: Let the missing number is “a”. We know that, Product of
means =Product of extremes.
Therefore 3 × a = 4 × 12; By dividing both sides by 3, we get the missing term
= (4 × 12)/3 = 16
Example 8: Taking 4 and 16 are means, write any two proportions.
Sol: Given 4 and 16 are means. So, : 4 = 16:
The product of Means is 4 × 16 = 64. Hence the product of Extremes
mustalso be 64
64 can be written as 4 × 16 or 2× 32 etc. Two proportions are 2: 4:: 16 : 32
and16 : 4 :: 16 : 4.

• FOURTH PROPORTIONAL:
If a : b = c : d, then d is called the fourth proportional to a, b, c.

Example 9: Find the fourth proportional of the numbers 12, 48, 16.
Sol: Let fourth proportional is x. Now as per the concept above the product
ofextremes should be equal to the product of the means → 12/48 = 16/x
→ x = 64.

• THIRD PROPORTIONAL: a : b = c : d, then c is called


the thirdproportion to a and b.

Example 10: If 2, 5, x, 30 are in proportion, find the third proportional “x”.


Sol: Here x is third proportional. According to the concept 2/5 = x/30 → x = 12.

• MEAN PROPORTIONAL: Mean proportional between a and b is √ab .

Example 11: Find the mean proportional of the numbers 10 and 1000.
Sol: Mean proportional between a and b is √ab. Let the mean proportional
of10 and 1000 be x.
So x = √10x1000 = √10000 = 100.

• CONTINUED PROPORTION a, b, c are in Continued


11
Proportion if a : b = b : c. Here b is called the Mean
Proportional and is equal to the square root of the product of
a and c.b2 = a × c → b = √aca/b = b/c = c/d etc., then a, b, c,
d are in Geometric Progression. Let a/b = b/c = c/d = k, then,
c = dk; b = ck and a = bk
Since c = dk, b = dk × k = dk2 and a = bk = dk2 × k = dk3, implying
theyare in Geometric Progression.
If the three ratios, a : b, b : c, c : d are known, we can find a : d by
themultiplying these three ratios
a/d = a/b × b/c × c/d
• If a/b = c/d= e/f , then each of these ratios is equal to (a+c+e)/(b+d+f)
• If a/b = c/d, then b/a = d/c(Invertendo)
• If a/b = c/d, then a/c = b/d (Alternendo)
• If a/b = c/d , then (a+b)/b = (c+d)/d(Componendo)
• If a/b = c/d, then (a-b)/b = (c-d)/d (Dividendo)
• If a/b = c/d, then (a+b)/(a-b) = (c+d)/(c-d), (Componendo & dividendo)

Example 12: If a : b = 2 : 5, then find the value of (3a + 4b) : (5a + 6b).
Sol: Let a = 2x & b = 5x. Then (3a + 4b): (5a + 6b) = (3 × 2x + 4 × 5x)
: (5 ×2x + 6 × 5x) → 26x:40x = 13 : 20.

• DIRECT VARIATION Two quantities “x” and “y” are said to be


in directvariation if an increase in one quantity results in increase
in the other quantity and decrease in one results in decrease in the
other quantity. If two quantities vary always in the same ratio, then
they are in direct variation.

Examples for Direct Variation:

1. Distance and Time are in Direct Variation, because more the


distancetravelled, the time taken will be more (if speed remains the
same).
2. Principal and Interest are in Direct Variation, because if the
Principal ismore, the Interest earned will also be more.
3. Purchase of Articles and the amount spent are in Direct Variation,
because purchase of more articles will cost more money. If two
quantities “x” and “y” vary directly in such a way that x/y remains
constant and is positive, and this constant is called the constant of
variation. If x α y that means x = py where p is proportionality
constantx/y = p, then ratio of any two values of “x” is equal to the
ratio of corresponding values of “y” Then x1/x2 = x2/y2.

Example 13: Sam takes 2 hours to cover 40 km. Find the distance he
willtravel in 8 hours.
Sol: Let distance covered = y. When time increases the distance also
increases. Therefore, they are in direct variation, 2 : 8 = 40 : y → y = (40
12
×8)/2 = 160 km. Sam will travel 160 km in 8 hours.
Example 14: The purchase price of 15 articles is Rs 4500. Find number
ofarticles purchased for Rs. 1500.
Sol: Let articles purchased = x. When amount spent decreases, then
numberof articles also decreases. So they are in direct variation → 15 : x
= 4500 : 1500 → x = (15 × 1500) / 4500 = 5
Example 15: The cost of 10 kg sugar is Rs 360. Find the cost of 18.5
kgsugar.

13
Sol: Let the cost is Rs. X. When quantity increases, cost also increases.
Sothey are in direct variation → 10/18.5 = 360/X → X = 666

• INVERSE VARIATION:
If two quantities “x” and “y” are such that an increase or decrease
in “x”leads to a corresponding decrease or increase in “y” in the
same ratio, then we can say they vary indirectly or the variation is
inverse.
Suppose 6 men can do a piece of work in 18 days, then 12 men can
dothe same job in 9 days. That means if we double the number of
men, then number of days get halved. That means there is inverse
relation between number of men and number of days.
In general, when two variables x and y are such that xy = k where k
is a non-zero constant, we say that y varies inversely with x. In
notation, inverse variation is written as y α 1/x → y = p/x, where p
is constant of proportionality → xy = p. So x1y1 = x2y2.

Examples for Inverse Variation:

1. Work and Time are in Inverse Variation, because more the number
ofthe workers, lesser will be the time required to complete a job.
2. Speed and Time are in Inverse Variation, because higher the
speed,the lower is the time taken to cover a distance.
3. Population and Quantity of food are in Inverse Variation, because if
thepopulation increases, the food availability decreases.

Example 16: Suppose that y varies inversely as x and that y = 12 when x = 6.


a) Form an equation connecting x and y.
b) Calculate the value of y when x = 18.
Sol: x and y are in inverse proportion. So x1y1= x2y2 → 6 × 12 = 18 × y
→ y =4

PRACTICE QUESTIONS:

Example 1: Are the ratios 4:5 and 8:10 said to be in Proportion?

Solution:

4:5= 4/5 = 0.8 and 8: 10= 8/10= 0.8

Since both the ratios are equal, they are said to be in proportion.

Example 2: Are the two ratios 8:10 and 7:10 in proportion?


14
Solution:

8:10= 8/10= 0.8 and 7:10= 7/10= 0.7

Since both the ratios are not equal, they are not in proportion.

Example 3: Given ratio

are-a:b = 2:3

b:c = 5:2

c:d = 1:4

Find a: b: c.

Solution:

Multiplying the first ratio by 5, second by 3 and third by 6, we

havea:b = 10: 15

b:c = 15 : 6

c:d = 6 : 24

In the ratio’s above, all the mean terms are equal,

thusa:b:c:d = 10:15:6:24

Example 4: Check whether the following statements are true or

false.a] 12 : 18 = 28 : 56

b] 25 people : 130 people = 15kg : 78kg

Solution:

a] 12 : 18 = 28 : 56

The given statement is false. 12 : 18 = 12 / 18 = 2 / 3 = 2:3 28 : 56 =

28 / 56 = 1 / 2 = 1 : 2

15
They are unequal.

b] 25 persons : 130 persons = 15kg :

78kgThe given statement is true.

25 people : 130 people = 5: 26

15kg : 78kg = 5: 26

They are equal.

Example 5: The earnings of Rohan is 12000 rupees every month and


Anish is 191520 per year. If the monthly expenses of every person are
around 9960rupees. Find the ratio of the savings.

Solution:

Savings of Rohan per month = Rs (12000-9960) = Rs.

2040Yearly income of Anish = Rs. 191520

Hence, the monthly income of Anish = Rs. 191520/12 = Rs. 15960.

So, the savings of Anish per month = Rs (15960 – 9960) = Rs. 6000

Thus, the ratio of savings of Rohan and Anish is Rs. 2040: Rs.6000 = 17: 50.

Example 6: Twenty tons of iron is Rs. 600000 (six lakhs). What is the
cost of560 kilograms of iron?

Solution:

1 ton = 1000 kg
20 tons = 20000 kg
The cost of 20000 kg iron = Rs. 600000
The cost of 1 kg iron = Rs{600000}/
{20000}
= Rs. 30
The cost of 560 kg iron = Rs 30 × 560 = Rs 16800

Example 7: The dimensions of the rectangular field are given. The length
andbreadth of the rectangular field are 50 meters and 15 meters. What is
the ratioof the length and breadth of the field?
16
Solution:

Length of the rectangular field = 50 m

Breadth of the rectangular field = 15 m

Hence, the ratio of length to breadth = 50:

15

⇒ 50: 15 = 10: 3.

Thus, the ratio of length and breadth of the rectangular field is 10:3.

Example 8: Obtain a ratio of 90 centimetres to 1.5 meters.


Solution:

The given two quantities are not in the same

units.Convert them into the same units.

1.5 m = 1.5 × 100 = 150 cm

Hence, the required ratio is 90 cm: 150 cm

⇒ 90: 150 = 3: 5

Therefore, the ratio of 90 centimetres to 1.5 meters is 3: 5.

Example 9: There exists 45 people in an office. Out of which female


employees are 25 and the remaining are male employees. Find the ratio of

a] The count of females to

males.b] The count of males to

females.Solution:

Count of females = 25

Total count of employees = 45 Count of males = 45 – 25= 20

The ratio of the count of females to the count of males

17
= 25 : 20

=5:4

The count of males to the count of females

= 20 : 25

=4:5

Example 10: Write two equivalent ratios of 6: 4.

Answer:

Given Ratio : 6: 4, which is equal to 6/4.

Multiplying or dividing the same numbers on both numerator


anddenominator, we will get the equivalent ratio.

⇒(6×2)/(4×2) = 12/8 = 12: 8

⇒(6÷2)/(4÷2) = 3/2 = 3: 2

Therefore, the two equivalent ratios of 6: 4 are 3: 2 and 12: 8

Example 11: Out of the total students in a class, if the number of boys
is 5 and the number of girls is 3, then find the ratio between girls and
boys.

Solution:

The ratio between girls and boys can be written as 3:5 (Girls: Boys). The
ratiocan also be written in the form of factor like 3/5.

Example 12: Two numbers are in the ratio 2 : 3. If the sum of numbers is
60,find the numbers.

Solution:

Given, 2/3 is the ratio of any two

numbers.Let the two numbers be 2x and

3x.

18
As per the given question, the sum of these two numbers =

60So, 2x + 3x = 60

5x = 60

x = 12

Hence, the two numbers

are;2x = 2 x 12 = 24

and

3x = 3 x 12 = 36

24 and 36 are the required numbers.

19
2. PERCENTAGE:

( 1) The term percent comes from the Latin phrase 'per centum' which
meansper hundred or for every hundred. It is a fraction whose denominator
is 100

and numerator is percent, e.g. 40% or . In mathematics, percent is


denotedby the symbol '%'.

2) How to convert a fraction into a percent: To convert a fraction into

percentmultiply it by 100, e.g.

3) How to convert a percent into a fraction: Divide the number by 100 and

drop the percent symbol, e.g. 60%

4) The percentage of a given number 'n' is given

by;x % of a given number 'n' =

E.g. 70% of 200 = * 200 = 140

Some quicker methods:

1) If two values are respectively x% and y% more than a third value, the

firstvalue is % of the second value.

And, the second value is % of the first value.

2) If two values are respectively x% and y% less than a third value, the

firstvalue is % of the second value.

And, the second value is % of the first value.

3) If the price of a commodity increases by x %, the reduction in


consumptionso as not to increase the expenditure is given by;

20
4) If the price of a commodity decreases by x %, the increase in consumption
soas not to decrease the expenditure is given by;

5) If A is x% of C and B is y% of C, A would be * 100 % of B.

6) Percentage fraction table: Some important fractions to remember

5) x % of a quantity is taken by A, y % of the remaining is taken by B and


z %of the remaining is taken by C. If P is left in the fund, there was

in the beginning.

6) x % of a quantity is added, y% of the increased quantity is added, again z


% of the increased quantity is added and it becomes A, the initial amount
isgiven by;

7) The population of a town is P. If it increases by x % in the first year, y


% in the second year and z% in the third year, the final population after
three yearsis given by;

And, if the population decreases by y % in the second year, the


populationafter three years is given by;

21
Similarly, if the present population of a city changes (increases or
decreases)at r % per annum, the population after n years is given by;

And, the population n years ago is given by;

Note: Use '+' sigh if the population is increasing at r % per annum and
use '-'sign if it is decreasing at r % per annum.

8) If a number is r % more than the second number, the second number

willbe % less than the first number, e.g. If A's income is r %

more

than B's income, B's income is % less than A's income.

9) If a number is r % less than the second number, the second number will be

% more than the first number.

10) If a value is increased by x % and later decreased by x %, net change

inthe value is always a decrease which is equal to x % of x or .

11) If a value is first increased by x %, decreased by y%, there will be

% increase or decrease in the value, i.e. '+' sign will show


anincrease and '- 'sign will show a decrease in the value.

12) If a value is increased by x % and y % successively, the final increase


inthe value is given by;

13) If the price of a product is reduced by x % and its consumption


is increased by y % or the price is increased by x % and
consumption isdecreased by y%, the effect on revenue is given by;

= percent increase - percent decrease =


22
'+' sign will show an increase and '-' sign will show a decrease in the value.

14) The pass marks in an examination are x %. If a student secures y


marksand fails by z marks, the maximum marks are given by;

15) A candidate scores x % marks in an examination and fails by 'a' marks.


Ifanother candidate who scores y % marks which is 'b' marks more than
the required pass marks, the maximum marks for this examination are
given by;

16) The sides of a triangle are measured. If one side is taken x % in


excessand the other side is taken y% in deficit, the error percent in area
calculatedfrom these measurements is given by;

'+' sign will show the excess and '-' sign will show the deficit in the area.

17) If the sides of a triangle, rectangle, square or any other two-

dimensionalshape are increased by x %, the area is increased by

18) In an examination, x% students failed in one subject and y%


students failed in another subject. If z% students failed in both the
subjects, the percentage of students who passed in both the subjects is
given by;

= 100 - (x + y - z)

23
PRACTICE QUESTIONS:

Q1. A student obtained 62.5 % marks in a certain examination. If


themaximum marks are 800. Find the total marks obtained by her.
Solution: 800 * 62.5/100 = 500

Q2. The annual sales of company B were Rs 96,000 in fiscal year 95-96
andRs 1,08,000 in fiscal 96-97. What was the % increase in turnover?
Solution: Increase = 96,000 – 1,08, 000 = 12,000
∴ % Increase = 12000 * 100/96000 = 100 * 1/8 = 12.5 %

Q3. The price of a Bajaj Scooter is Rs 25,000 which is 20 % lesser


than aLML Scooter. What is the price of a LML Scooter?
Solution: Here note that the % given is defined in terms of the price of
LML Scooter and not Bajaj Scooter. ∴ Computing the price of SMS as
1.2 times Bajaj's price will give incorrect answer. The data given is:
LML * (100 – 20)/100 = Bajaj = 25,000
∴ LML =25,000 × 100/80 = Rs 31,250
Or you can simply do = 25,000/0.8 = 31250
(20 % less means it is 0.80 times of the other one)

Q4. The price of rice increases by 30 %. In order to keep the expenses on


riceconstant as before, by what percentage should a person cut down his
consumption?

Solution: Applying the formula 100 × R/ (100 + R), Since the price
hasincreased, the consumption should be reduced by:
(100 × 30)/ (100+30) = 23 1/13 % = 300/13 %. Hence option C

Q5. Tom’s income is 20 % less than Jerry’s. How much is Jerry’s


incomemore than Tom’s?
Solution: Applying the formula 100 * R/(100
- R)As , income is 20% less
Jerry’s income is more by = 100 * 20/80 = 25 %

Q6. A traveling salesman carried 75 % of his money in traveler’s cheque


and25 % in cash. During one of his journeys, he lost his entire cash and
spent from his traveler’s cheque. On completion of the journey, he
returned 30% ofthe traveler’s cheque, which amounted to Rs 180. What
was the total money that he carried?
Solution: 30 % of Traveler’s Cheque = Rs 180
∴ 100 % of Traveler's Cheque = 180 * 100/30 = Rs 600
Since Traveler's Cheque accounted for 75% of the total money that he
hadcarried, the total money that he carried is: 600 * 100/75 = Rs 800

24
Q7. A number is increased by 20%, and then it is decreased by 30%, what
isthe net change in the number?
Solution: Using the formula: X + Y + XY/100
⇒ 20 + (-30) + 20 *-30 /100 = -16
This means there is a decrease of 16 %

Q8. A square is converted into a rectangle by increasing one of its sides by 5


%and reducing the other by 5 %. What will be the % change in the area of the
two figures?
Solution: Let the side of the square = a
∴ Its area = a2; When the square is being converted to a rectangle, the
lengthbecomes 1.05a and the width becomes 0.95a. ∴ New area = 1.05a ×
0.95a =0.9975a2
Change in area = a decrease of 0.0025a2
∴ % decrease in area =0.0025a2 * 100/a2= 0.25 %
Q9. In an election there were two candidates P and S. The poll turnout
wasonly 90 %. 500 of the given votes were declared invalid. P won 440
of X = 400 % of the total votes. Find the total number of eligible voters
and the number of votes received by each of the candidates.
Solution: Let the registered voters =
XP got 440/9 % of X = 4.4X/9
Since P got 400 votes more than S, S got 4.4X/9 –
400Total votes registered = X = P + S + Invalid votes
X = 4.4 X/9 + (4.4 X/9 – 400)+ 500 ⇒ X = 8.8X/9 + 100
⇒ X = 4500. Since the poll turnout was only 90%, the total number of
eligiblevoters = 100/90 * 4500 = 5000
Number of votes P got = 4.4/9 * 4500 =
2200 Number of votes S got = 2200 – 400
= 1800.

Q10. If the population of a town is 926100 and it has been growing


annually at5 %, what was the population 3 years ago?
Solution: Let the population be X
X * (100 + 5)3 / (100) 3 =926100; X * (105/ 100) * (105/100) * (105/100) =
926100
⇒ X * (21/20) * (21/20) * (21/20) = 926100
⇒ X = 926100 * (20/21) * (20/21) * (20/21) = 800,000

Q11. Two friends, Akash & Beenu had some candies each. One of them
had 15 candies more than the other. The candies with Akash was 60% of
the totalcandies with them. How many candies did each have?
Explanation Let the candies with be (x + 15) and
x.Therefore, x + 15 = 60/100(x + 15 + x)
(x + 15) = 3/5(2x +
15)5x + 75 = 6x + 45
x = 30
So, the marks of two students are 45 and 30.
25
Q12. A fruit seller had some oranges. He sells 30% oranges and still has
140mangoes. Originally, he had:
Explanation: Suppose originally he had x
oranges.Then, (100 - 30)% of x = 140.
70/100 x = 140
x = (140 x 100)/70 = 200.

Q13. What percentage of numbers from 1 to 30 has 1 or 9 in the unit's digit?


Explanation: Such numbers from 1 to 30 are 1, 9, 11, 19, 21, 29
Number of such numbers =6
Required percentage is (6/20 * 100) % = 20%
Q14. If M = y% of z and N = z% of y, then which of the following must be true?
Explanation: y% of z = (y/100 x z) = (z/100 x y) = z%
of yM = N.

Q15. In ABC College, 65% of students are less than 20 years of age. The
number of students more than 20 years of age is 2/3 of number of students
of20 years of age which is 42. What is the total number of students in the
ABC College?
Explanation: Let total number of students is x.
Then, number of students more than or equal to 20 years of age = (100 - 65)%
of x = 35% of x.
As per the question, 35% of x = 42 + 2/3 of 42. So 35x/100 = 70. Hence
x =200.

Q.16. A student erroneously multiplied a number by 2/5 instead of


5/2.What isthe percentage error in the calculation?
Explanation: Let the number be 100. 2/5 of 100 is 40 while 5/2 of 100 is
250.Now the difference is 210 on a base of 250. Therefore, percentage
differenceis 210/250 into 100 = 84%.

Q.17. The sum of the first six terms of an AP is 48 and the common
differenceis 2. What is the 4th term?
Explanation: Let the 1st number of an A.P is x,
As the sum of 1st 6 numbers is 48, so x+ x+2+x+4+x+6+x+8+x+10 =
48, sox= 3
So 4th term is 9.

Q18. Three candidates, Ajay, Bijoy & Chandu contested an election and
received 1800, 3300 and votes 3900 respectively. What percent of the total
votes did A get?
Explanation: Total no. of votes polled = (1800 + 3300 + 3900) =
9000.Required percentage = (1800/9000 * 100)% = 20%.
Suggested Action:

26
Q19. A Stationery seller had some Pens, Sharpeners, Erasers & Pencils.
Hesells 65% of the total units and still has 175 units. Originally, he had:
Explanation: Suppose originally he had x units.
Then, (100 - 65)% of x =
175.35/100 x = 175
x = 500

Q20. The total population of a village increased from 1,80,00 to 22, 500
in adecade. The average percentage increase of population per year of
that village is:
Explanation: Population increase in 10 years = (22500- 18000) =
4500.Increase% = (4500/18000 x 100)% = 25%
Required average = (25/10)% = 2. 5

27
PARTNERSHIP

⚫ What Is a Partnership?

A partnership is a formal arrangement by two or more parties to manage


andoperate a business and share its profits.

There are several types of partnership arrangements. In particular, in a


partnership business, all partners share liabilities and profits equally, while
in others, partners may have limited liability. There also is the so-called
"silent partner," in which one party is not involved in the day-to-day
operations of thebusiness.

⚫ Features of Partnership:

Following are the few features of a partnership:

1. Agreement between Partners: It is an association of two or more


individuals, and a partnership arises from an agreement or a
contract. The agreement (accord) becomes the basis of the
association betweenthe partners. Such an agreement is in the written
form. An oral agreement is evenhandedly legitimate. In order to
avoid controversies, it is always good, if the partners have a copy of
the written agreement.
2. Two or More Persons: In order to manifest a partnership, there
should be at least two (2) persons possessing a common goal. To put
itin other words, the minimal number of partners in an enterprise can
be two (2). However, there is a constraint on their maximum number
of people.
3. Sharing of Profit: Another significant component of the
partnership is, the accord between partners has to share gains and
losses of a trading concern. However, the definition held in the
Partnership Act elucidates – partnership as an association between
people who have consented to share the gains of a business, the
sharing of loss is implicit. Hence, sharing of gains and losses is
vital.
4. Business Motive: It is important for a firm to carry some
kind ofbusiness and should have a profit gaining motive.
5. Mutual Business: The partners are the owners as well as the
agent of their firm. Any act performed by one partner can affect
other partners and the firm. It can be concluded that this point acts
as a test of partnership for all the partners.
28
2. 6. Unlimited Liability: Every partner in a partnership has unlimited
liability.

⚫ Types of Partnerships

A partnership is divided into different types depending on the state and


wherethe business operates. Here are some general aspects of the three
most common types of partnerships.

• General Partnership

A general partnership comprises two or more owners to run a business. In


this partnership, each partner represents the firm with equal right. All
partners can participate in management activities, decision making, and
have the right to control the business. Similarly, profits, debts, and
liabilities are equally shared and divided equally.

In other words, the general partnership definition can be stated as those


partnerships where rights and responsibilities are shared equally in terms
of management and decision making. Each partner should take full
responsibility for the debts and liability incurred by the other partner. If
one partner is sued, all the other partners are considered accountable. The
creditor or court will hold the partner’s personal assets. Therefore, most of
thepartners do not opt for this partnership.

• Limited Partnership

In this partnership, includes both the general and limited partners. The
general partner has unlimited liability, manages the business and the
otherlimited partners. Limited partners have limited control over the
business (limited to his investment). They are not associated with the
everyday operations of the firm.

In most of the cases, the limited partners only invest and take a profit
share.They do not have any interest in participating in management or
decision making. This non-involvement means they do not have the right
to compensate the partnership losses from their income tax return.

• Limited Liability Partnership

In Limited Liability Partnership (LLP), all the partners have limited


liability. Each partner is guarded against other partners legal and financial
mistakes. A limited liability partnership is almost similar to a Limited
29
Liability Company (LLC) but different from a limited partnership or a
general partnership.

• Partnership at Will

Partnership at Will can be defined as when there is no clause mentioned


about the expiration of a partnership firm. Under section 7 of the Indian
Partnership Act 1932, the two conditions that have to be fulfilled by a
firm tobecome a Partnership at Will are:

• The partnership agreement should have not any fixed expiration date.
• No particular determination of the partnership should be mentioned.

Therefore, if the duration and determination are mentioned in the


agreement,then it is not a partnership at will. Also, initially, if the firm had
a fixed expiration date, but the operation of the firm continues beyond the
mentioneddate that it will be considered as a partnership at will.

⚫ Profit and/or loss is shared among the partners based on the sum
ofmoney invested by individual partners and the time period of the
investment.

For example: A partner who has invested the highest amount of


money/resources will receive the highest share of the profit at the end
of the year provided all the partners have invested for the same time
period.

Ratio of Division of Gains:

1. Suppose A and B invest Rs. x and Rs. y respectively for a year


in abusiness, then at the end of the year:

(A’s share of profit) : (B’s share of profit) = x : y

Here investment of all partners are for same time, and the gain or loss
isdistributed among them in the ratios of their investments.

2. Suppose A invests Rs. x for ‘p’ months and B invests Rs. y for ‘q’
months,then

(A’s share of profit) : (B’s share of profit) = xp : yq

30
Here investments are for different time periods, equivalent capitals
arecalculated for a unit of time by taking,

(capital x number of units of time).

Profit or loss is divided in the ratio of these capitals.

Working partner: The partner one who works for the business is
called aworking partner.

Sleeping partner: The partner who simply invests the money for the
businessand doesn’t work is called a sleeping partner.

Concept 1: If a group of n persons invested different amount for


differentperiod then their profit is the ratio is At1 : Bt2 : Ct3 : Dt4 :
…… : Xtn

Here first person invested amount A for t1 period, second persons


investedamount B for t2 period and so on.

Example 1:
A starts a business with Rs 2,000, B joins him after 3 months with Rs 4,000.
Cputs a sum of Rs 10, 000 in the business for 2 months only. At the end of
the year the business gave a profit of Rs 5600. How should the profit be
divided among them?

Solution:

Ratio of their profits (A’s : B’s : C’s) = 2000 x 12 : 4000 x 9 : 10000 x 2


= 24000:36000:20000
=6:9:5

Now, 6 + 9 + 5 = 20

Then
A’s share = 5600× 𝟔 = Rs 1680
𝟐𝟎

B’s share = 5600 × 𝟗 = Rs 2520


𝟐𝟎

31
C’s share = 5600 × 𝟓 = Rs 1400
𝟐𝟎

Concept 2: If investments are in the ratio of a : b : c and the timing of


their investments in the ratio of x : y : z then the ratio of their profits are
in the ratioof ax : by : cz.

Example 2:
A, B and C invested capital in the ratio 2 : 3 : 5, the timing of their
investmentsbeing in the ratio 4 : 5 : 6. In what ratio would their profit be
distributed?

Solution:

We should know that If the duration for their investments be in the


ratiox : y : z, and investments is in ratio a : b : c then the profit would
be distributed in the ratio ax : by : cz .

Thus, following the same rule, the required ratio


= 2 x 4 : 3 x 5 : 5 x 6 = 8 : 15 : 30

Concept 3: If investments are in the ratio a : b : c and profits in the ratio p


: q :r, then the ratio of time is 𝑝 : 𝑞 : 𝑟.
𝑎 𝑏 𝑐

Example 3:
A, B and C invested capital in the ratio 5 : 6: 8. At the end of the business
term, they received the profits in the ratio 5 : 3 : 12. Find the ratio of time
forwhich they contributed their capital?

Solution:

Using the above formula, we have the required ratio

𝟓 : 𝟑 : 𝟏𝟐
=𝟓 𝟔 𝟖

=1:𝟏:𝟑
𝟐 𝟐

=2:1:3

32
Practice Questions

1. A invested 125% as much money as B, C invested 80% as much


money as B. The total of all the three is Rs. 61,000. How much did
C invest?

Solution :

Because both A and C are compared to B, let us assume the investment of


Bas "x".

Given : A invested 125% as much money as B.

Investment of A = 125% of x = 1.25x

Given : C invested 80% as much money as B.

Investment of C = 180% of x = 0.8x

Given : The total of all the three is

Rs.61,000So, we have

1.25x + x + 0.8x = 61000

3.05x = 61000

Divide both sides by 3.05

x =

61000/3.05x =

20000

Investment of C is

= 0.8x

= 0.8 × 20000

= 16000
33
So, C invested Rs.16000.

2. Daniel started a business with a capital of Rs. 8000. After six


months, David joined him with investment of some capital. If at the
end of the year, each of them gets equal amount as profit, how much
did David invest?

Solution:

Let "x" be the investment of David.

David's investment was in the business for six months.

Daniel invested Rs.8000 and his investment was in the business for 12
months.

Then, the profit sharing ratio of Daniel and David is

8000×12 : x × 6

Given : At the end of the year, each of them gets equal amount as profit.

So, we have

8000×12 : x × 6 = 1:1

96000: 6x =1:1

96000= 6x

x = 16000

So, David's invested Rs.16,000.

3. A, B and C start a partnership.The capitals of A, B and C are in


theratio 10 : 9 : 6 and the time period of A and B is in the ratio 2 : 3.
B getsRs.10,800 as his share out the of a total profit of Rs.26,000. If
A's capital was there is in the business for 8 months, for how many
monthswas C's capital in the business ?

Solution :

34
Let C's capital be in the business for "x"

months.Ratio of capitals is

A : B : C = 10 : 9 : 6

Ratio of time period is

A:B =2:3

A's capital was there is in the business for 8 months

,Then ,

A:B = 8 : 12

A:B:C = 8:12:x

From the above ratio, we have

A's share = 8 ×10 = 80

B's share = 12 × 9 = 108

C's share = x × 6 = 6x

Given : B's share is Rs.10,800.and total profit of Rs.26,000

Then, we
have
×26000 = 10800
𝟏𝟎
𝟖
𝟏𝟖𝟖 + 𝟔𝒙

2808000 = 10800(188 + 6x)

28080= 108(188+ 6x)

35
28080= 20304 + 648x

28080- 20304= 648x

7776/ 648 =

x X = 12

So, C's capital was in the business for 12 months.

4 . A and B start a partnership by investing Rs 24,000 and Rs. 36,000


respectively. Their agreement is to share half of the total profit equally and
then share the remaining half in the ratio of their capital. If they share the
entire profit in the ratio of their capitals, B would have got Rs. 2500 more
than what she would have got otherwise. What is the total profit?

Solution :

Given : A and B start a partnership by investing Rs.24,000 and


Rs.36,000respectively.

Ratio of capitals A and B is

= 24000 : 360000

= 2:3

Let the total profit be "x"

Half of the total profit = x / 2.

Given : A and B share half of the total profit

equally.Then,

B's share = (x/2) / 2 = x / 4

Given : A and B share remaining half of the total profit in the ratio of
theircapitals.

Ratio of their capitals is 2 :

36
3.So, we have

B's share = 3/5 × x/2 = 3x / 10

Total profit of B is

= x/4 + 3x/10

= 5x/20 + 6x/20

= 11x / 20 --- (1)

Sharing the entire profit "x" in the ratio of their capitals (That is 2 : 3) :

B's share = 3x / 5 --- (2)

Given : In (2), B would get Rs.2500 more than what she gets in

(1).So, we have

3x/5 - 11x/20 = 2500

12x/20 - 11x/20 = 2500

x/20 = 2500

x = 50000

So, the total profit is Rs. 50,000

5. Two friends A and B started a business with initial capital


contribution of Rs.1 lac and Rs. 2 lacs. At the end of the year, the business
made a profit of Rs. 30,000. Find the share of each in the profit.

Solution : We know that if the time period of investment is same,


profit/loss isdivided in the ratio of value of investment.
=> Ratio of value of investment of A and B = 1,00,000 : 2,00,000 = 1 : 2
=> Ratio of share in profit = 1 : 2
=> Share of A in profit = (1/3) x 30,000 = Rs. 10,000
=> Share of B in profit = (2/3) x 30,000 = Rs. 20,000

6. Three friends A, B and C started a business, each investing Rs. 10,000.


After 5 months A withdrew Rs. 3000, B withdrew Rs. 2000 and C invested
Rs. 3000 more. At the end of the year, a total profit of Rs. 34,600 was
recorded. Find the share of each.
37
Solution : We know that if the period of investment is not uniform, the
gains/losses from the business are divided in the ratio of their inputs,
where input is calculated as the product of amount of investment and time
period ofinvestment.
So, input = value of investment x period of investment, and here, period
of investment would be broken into parts as the investment is not
uniform throughout the time period.
A’s input = (10,000 x 5) + (7,000 x 7) = 99,000
B’s input = (10,000 x 5) + (8,000 x 7) = 1,06,000
C’s input = (10,000 x 5) + (13,000 x 7) = 1,41,000

=> A : B : C = 99000 : 106000 : 141000


=> A : B : C = 99 : 106 : 141
=> A : B : C = (99 / 346) : (106 / 346) : (141 / 346)
Thus, A’s share = (99 / 346) x 34600 = Rs. 9900
B’s share = (106 / 346) x 34600 = Rs. 10600
C’s share = (141 / 346) x 34600 = Rs. 14100

7. A invested Rs. 70,000 in a business. After few months, B joined him


with Rs. 60,000. At the end of the year, the total profit was divided
between themin ratio 2 : 1. After how many months did B join?

Solution : Let A work alone for ‘n’ months.


=> A’s input = 70,000 x 12
=> B’s input = 60,000 x (12 – n)
So, (70,000 x 12) / [60,000 x (12 – n)] = 2 / 1
=> (7 x 12) / [6 x (12 – n)] = 2 / 1
=> 12 – n = 7
=> n = 5
Therefore, B joined after 5 months

8. A, B and C started a business by investing Rs. 1,20,000, Rs.


1,35,000and Rs. 1,50,000 respectively. Find the share of each, out of
an annual profit of Rs. 56,700.

Solution:

Ratio of shares of A,B and C = Ratio of their investments

= 120000 : 135000 : 150000

= 8 : 9 : 10.

38
9. Alfred started a business investing Rs. 45,000. After 3 months,
Peterjoined him with a capital of Rs. 60,000. After another 6 months,
Ronaldjoined them with a capital of Rs. 90,000. At the end of the
year, they made a profit of Rs. 16,500. Find the share of each.

Solution:

Clearly, Alfred invested his capital for 12 months, Peter for 9 months and
Ronald for 3 months.

So ratio of their capitals = (45000×12) : (60000×9) :


(90000×3)

= 540000 : 540000 : 270000 = 2 : 2 : 1

Total profit is Rs.

16500 So,their profit

share

Alfred’s Share = 16500 × 2/5

=6600 Peter’s Share = 16500 × 2/5

=6600 Ronald’s Share = 16500

×1/5=3300

10. A, B and C start a business each investing Rs. 20,000. After


5 months A withdrew Rs.5000, B withdrew Rs. 4000 and C invests
Rs.6000 more. At the end of the year, a total profit of Rs. 69,900
was recorded. Find the share of each.

Solution:

Ratio of the capitals of A, B and C

=(20000×5+15000×7) : (20000×5+16000×7): (20000×5+26000×7)

= 205000 : 212000 : 282000


39
= 205 : 212 : 282.

Total profit of Rs.

69,900So,

Profit share of A =69900 ×205 / 699 =

Rs.20500 Profit share of B=69900 × 212 / 699

= Rs.21200 Profit share of C= 69900 × 282 /

699 = Rs.28200

11. A, B and C enter into partnership. A invests 3 times as much as B


invests and invests two-thirds of what C invests. At the end of the
year,the profit earned is Rs. 6600. What is the share of B?

Solution:
Let C’s capital = Rs. x. Then, B, capital = Rs.
A’s capital = Rs, 3 × 𝟐 x
𝟑

= Rs.

2x Their profit

share,

A: B: C = 2x : 2x : x
3

Total share = 11 x
3

profit earned is Rs. 6600

𝟐𝐱
So, Share of B = 6600 × 𝟑
𝟏𝟏 𝐱
𝟑

= Rs. 1200

40
Share of A = 6600 × 𝟐𝐱
𝟏𝟏 𝐱
𝟑

= Rs. 3600

Share of C = Rs. 1800


12. Four milkman rented a pasture. A grazed 24 cows for 3 months, B
10cows for 5 months; C 35 cows for 4 months and D 21 cows for 3
months.If A’s share of rent is Rs. 720, find the total rent of the field.

Solution:

Let the total rent of the field be x

Ratio of shares of A, B, C, D = (24×3) : (10×5) : (35×4) : (21×3)

= 72 : 50 : 140 : 63.

A’s share = Rs. 72 x.


325

Then ,
72
x = 720
325

X = 3250

Hence, total rent of the field is Rs. 3250.

13. Three persons started a business by investing Rs.6,00,000,


Rs.8,00,000, Rs.14,00,000 respectively. Calculate the share of
eachperson, out of annual profit of Rs.60,000?
Solution:

Given that

Ratios of their investments = 6,00,000 : 8,00,000 : 14,00,000

⇒ 6 : 8 : 14 ⇒ 3 : 4 : 7

Sum of the ratios = 3 + 4 + 7 = 14

Now, Share of first person = 60,000 x 3/14 =

41
12857.1Share of second person = 60,000 x 4/14 =

17142.9 Share of third person = 60,000 x 7/14 =

30000 Therefore, Share of each person is

Rs.12857.1, Rs.17142, Rs.30000 respectively.


14. Joseph, Johnson and Tom started a business each investing Rs.
20,000. After 5 months Joseph withdrew Rs.5000, Johnson withdrew
Rs.4000 and Tom invests Rs. 6000 more. At the end of the year, a total
profitof Rs. 69,900 was recorded. Find the share of each.

Solution:

Given that

Joseph, Johnson and Tom invested Rs.20,000

together and Tom invested more of Rs.6000 after 5

months

After 5 months, Joseph drew Rs.5000 and Johnson drew Rs.

4000Total profit of the year = Rs. 69,900

Therefore ratio of the capitals of Joseph, Johnson and Tom is

= (20000 x 5 months + 15000 x 7 months) : (20000 x 5 months +


16000 x 7 months) :( 20000 x 5 + 26000 x 7 months)

= 205000 : 212000 : 282000

= 205 : 212 : 282

Sum of the ratios of capitals = 205 + 212 + 282 = Rs. 699

Now, Joseph’s share = Rs.( 69900 x 205/699) = Rs.

20500; Johnson’s share = Rs.( 69900 x 212/699) = Rs.

21200; Tom’s share = Rs.( 69900 x 282/699) = Rs.

28200.

42
15. P, Q and R enter into partnership. P invests 2 times as much as
Qinvests and Q invests two- third of what R invests. At the end of
the year, the profit earned is Rs. 7000. What is the share of Q?

Solution:

Given that,

Profit earned by all of them at the end of the year = Rs.

7000Let the R’s capital be Rs. x

Then, Q’s capital = Rs.2/3x

P’s share = 2 x 2/3x = Rs.

4/3x

Therefore, Ratios of their capitals = 4/3x : 2/3x : x


= 4x : 2x : 3x

Sum of the ratios = 4 + 2 + 3 = 9

Then, Q’s share = Rs. (7000 x 2/9) = Rs. 1555.5


≅ Rs. 1556

16. A, B, and C bought a plot for Rs. 2 lakh. A contributed Rs. 1,50,000
when they sold that ,from the profit, B got Rs. 5050 while C got Rs.
3000.What was the profit of A?
Solution:

Given that,

Capital of A + capital of B + capital of C = Rs.

2,00,000Capital of A = Rs. 1,50,000

Then, Capital of (A + B + C) = Rs. 2,00,000

⇒ Capital of (B + C) = Rs. (2,00,000 – 1,50,000)

⇒ Capital of (B + C) = Rs. 50,000

Also given, Profit of (B + C) = Rs. (5050 + 3000) = Rs.


43
8050Therefore, A’s share = Rs. (8050 x 1,50,000/50,000)

⇒ Rs. 8050 x 3

⇒ RS. 24150

Therefore, profit of A = Rs. 24150

17. P and Q started a business with capitals in the ratio of 5 : 13. After
6months, Q took back his money. If they got profit in the ratio 25 :
26, forhow many months P’s capital continued in the business?

Solution:

Given that,

Let P continued the business for x months

Ratio of their capitals = 5 x : 13 x 6 ⇒ 5 x :

78Therefore,

Ratio of their capitals = ratio of the profit made by them

⇒ 5 x : 78 = 25: 26

⇒ 5 x x 26 = 78 x 25

⇒ 130x = 1950

⇒ x = 1950/130

⇒ x = 15 months

Hence, P invested his money for 15 months in the business.

P’s capital continued in the business for (x - 6) months i.e (15-6)months =


9months

44
UNIT 2:
BUSINESS MATHEMATICS
(Profit: Meaning, Working
examples
Loss: Meaning, Working
examples
Discount: Meaning, Types of
Discount, Working examples
Commission: Meaning, Types
of Commission agents,
Working examples
Brokerage: Meaning, Working
examples
Payroll: Meaning, Working
Examples)

45
PROFIT & LOSS
⚫ Cost Price: The price at which a particular article is bought is called
its cost price (C.P.).

⚫ Selling Price: Selling price (S.P.) is that at which a particular article


is sold.

⚫ Profit: If S.P. is more than C.P., then there is profit


Profit= (SP-CP)

⚫ Loss: If SP is less than CP then there has been a loss occurred.


Loss = (CP-SP)

IMPORTANT FORMULAE
1. Gain = (S.P.) - (C.P.)
2. Loss = (C.P.) - (S.P.)
3. Loss or gain is always calculated on C.P.
4. Gain Percentage: (Gain %)

Gain x 100
Gain % =
C.P.
5. Loss Percentage: (Loss %)

Loss x 100
Loss % =
C.P.
6. Selling Price: (S.P.)

(100 + Gain %)
SP = x C.P
100

46
7. Selling Price: (S.P.)

(100 - Loss %)
SP = x C.P.
100

8. Cost Price: (C.P.)

100
C.P. = x S.P.
(100 + Gain %)
9. Cost Price: (C.P.)

100
C.P. = x S.P.
(100 - Loss %)

Exercise :
Question 1: A student buys a pen for Rs 90 and sells it for Rs 100. Find his gain
and gain percent.
Solution:
C.P of pen = Rs.90
S.P of pen = Rs. 100
Gain = SP-CP
= 100-90 = 10
𝒈𝒂𝒊𝒏
Gain % =
𝑪𝑷
× 100
𝟏𝟎
= × 100
𝟗𝟎

=11.11%
Question 2: Rekha bought a saree for Rs.1240 and sold it for Rs. 1147. Find her
loss and loss percent.
Solution
C.P of saree = Rs. 1240
S.P of saree = Rs. 1147
47
Loss = CP - SP
= Rs (1240-1147)
= Rs. 93
𝟗𝟑
Loss% = ×100
𝟏𝟐𝟒𝟎
= 7.5 %

Question 3: A boy buys 9 apples for Rs. 9.60 and sells 11 apples for Rs.12. find
his gain or loss percent.
Solution
C.P of 9 apples = Rs. 9.60

CP of 1 apple = 9.60 / 9=1.067


S.P of 11 apple = Rs. 12

SP of 1 apple = 12 / 11=1.091
Clearly,
SP of 1 apple > CP of 1 apple
We get profit on selling apples = SP-CP = 1.091- 1.067
= 0.024

Gain% = 0.024 / 1.067 ×100


= 2.249 %
=2.25%
Question 4: The cost price of 10 articles is equal to the selling price of 9 articles.
Find the profit percentage.
Solution
Let the cost price of 1 article be Rs. C
Let the selling price of 1 article be Rs. S
Therefore, 10C = 9S

1 C= 9 / 10S
So the cost price is less than the selling price.
Profit = S.P-C.P

= S- 9 / 10S
= 1/ 10S
= 0.1 S
Profit% = gain/ CP × 100

48
= 11.11%

Question 5: A retailer buys a radio for Rs.225. his overhead expense are Rs15. if
he sells the radio for Rs.300, determine the profit percentage.

Solution
Radio cost = Rs 225
Overhead expenses = Rs 15
Total expenses = Rs.(225+15) = Rs.240
S.P = Rs.300
Profit = SP-CP = Rs (300-240) = Rs.60
Profit% = 60 / 240×100
= 25%
Question 6: A retailer buys a cooler for Rs.1200 and overhead expenses are on it
are Rs.40. if he sells the cooler for Rs.1550, Determine the profit percentage.
Solution
Cooler cost = Rs.1200
Overhead cost = Rs.40
Total cost = Rs.(1200+40) = Rs.1240
S.P of the cooler = 1550
Profit = S.P-C.P
= Rs.(1550-1240)
= Rs. 310

Profit% = 310 / 240×100


= 25%

Question 7: A dealer buys a wrist watch for Rs. 225 and spends Rs.15 on itsrepairs.
If he sells the same for Rs.300, find his profit percentage.
Solution
A dealer buys a wrist watch for Rs.225
Money spent on repairing the watch = Rs.15
Total expenses = Rs.(225+15) = Rs.240

49
S.P = Rs.300
Profit = SP-CP = Rs (300-240) = Rs.60
Profit% = 60 / 240×100
= 25%
Question 8
Ramesh bought two boxes for Rs.1300. he sold one box at a profit of 20% and the
other at a loss of 12%. if the selling price of both boxes are same, find the CP of
both boxes .
Solution
Let the cost price of the first box be Rs. x
Therefore, the cost of the second box will be Rs. (1300-
x)Profit on the first box = 20%
Loss on the second box = 12%

S.P of the first box = 𝟏𝟐𝟎 × x


𝟏𝟎𝟎
= 1.2 x
S.P of the second box = 𝟖𝟖 × (1300- x)
𝟏𝟎𝟎
= 0.88 x (1300 - x)
= 1144 - 0.88x
Selling prices of both the boxes are equal. So,

=> 1.2 x = 1144 - 0.88x


x= 550
The cost price of first box is Rs. 550
Cost price of the second box = Rs.(1300-550)
= Rs. 750
The cost prices of the 2 boxes are Rs.550 and Rs.750 respectively.
Question 9
If the selling price of 10 pens is equal to cost price of 14 pens, find the gain
percent?

Solution
Let the cost price of one pen be Rs. C

50
The selling price be Rs. S
Therefore, 10S = 14C

C = 10 / 14S
However, the cost price is less than the selling price.
Profit = SP - CP
=S - 10/ 14 S
= 4/14S

Profit % = 𝟒/𝟏𝟒 𝑺
× 100
𝟏𝟎 / 𝟏𝟒 𝑺

The required profit percentage is 40%.


Question 10
If the cost price of 18 chairs be equal to selling price of 16 chairs, find the gain or
loss percent
Solution
Let the cost price of one chair be Rs. C
Selling price be Rs. S
Therefore, 18C = 16S
C = 16 / 18 S
However, the cost price of the chair is more than that of selling price.
Profit = SP - CP
= S - 16/18 S
= 2/ 18 S
𝟐 / 𝟏𝟖 𝑺
Profit % = × 100
𝟏𝟔 / 𝟏𝟖 𝑺
= 2 / 16 × 100
= 12.5%

The required profit percent is 12.5%.


Question 11
If the selling price of 18 oranges is equal to the cost price of 16 oranges, find the
gain or loss percentage
Solution
S Let the cost price of one chair be Rs. C
Selling price be Rs. S
51
Therefore, 16 C= 18S
C = 18 / 16 S
However, the cost price of the chair is more than that of selling price.
Loss = CP - SP
= 18 / 16 S - S
= 2 / 16 S
Loss % = 𝟐 / 𝟏𝟔 𝑺 × 100
𝟏𝟖 / 𝟏𝟔 𝑺
= 11.11%

Question 12
Ravish sold his motorcycle to Vineet at a loss of 28%. Vineet spent Rs.1680 on it’s
repairs and sold the motorcycle to Rahul for Rs.35910, thereby making profitof
12.5%, find the cost price of the motorcycle for Ravish.
Solution
Let the cost price of the motor cycle for Ravish be Rs. x
Loss % = 28%
S.P = 72%x
Selling price of the motorcycle for ravish = cost price of the motorcycle for vineet
Money spent on repairs = Rs.1680
Therefore, total cost price of the motorcycle for vineet =Rs (0.72x + 1680)
Profit % = profit / CP × 100
𝟑𝟓𝟗𝟏𝟎 − (𝟎.𝟕𝟐𝐱 + 𝟏𝟔𝟖𝟎)
12.5% =
(𝟎.𝟕𝟐𝐱 + 𝟏𝟔𝟖𝟎)
𝟑𝟓𝟗𝟏𝟎 − (𝟎.𝟕𝟐𝐱 + 𝟏𝟔𝟖𝟎)
12.5 / 100 = (𝟎.𝟕𝟐𝐱 + 𝟏𝟔𝟖𝟎)

12.5 ×(𝟎. 𝟕𝟐𝐱 + 𝟏𝟔𝟖𝟎) = 100 ×(𝟑𝟓𝟗𝟏𝟎 − (𝟎. 𝟕𝟐𝐱 + 𝟏𝟔𝟖𝟎))


9x + 21000 = 3591000 - 72x - 168000
9x + 72x = 3591000-168000-21000
81 x = 3402000
= x= 42000
Ravish bought the motorcycle for Rs.42000
Question 13
By selling a book for Rs.258, a bookseller gains 20%. Find how much should he
sell to gain 30%?

52
Solution
Selling price of the book = Rs. 258
Gain = 20%

C.P = 100 / 120×258


= Rs. 215
S.P = 130 / 100×215
= Rs. 279.50
Therefore, the book seller must sell the book at Rs. 279.50 to make 30% profit.

Question 14
A defective briefcase costing Rs.800 is being sold at a loss of 8%. If the price is
further reduced by 5%, find its selling price?
Solution
C.P of the briefcase = Rs. 800
Loss = 8%
S.P = 92 /100×800
= Rs. 736
The price is decreased further by 5 %
S.P = 95 / 100×736
= Rs. 699.20
The selling price of the briefcase is Rs. 699.20

Question 15
By selling 90 ball pens for Rs160 a person loses 20%. How many ball pens should
be sold at Rs.96 so as to have a profit of 20%?
Solution
S.P of 90 ball pens = Rs 160
Loss= 20%
C.P = 100 / 80 ×160
= Rs. 200
Now,
S.P of x ball pens = Rs. 96

53
Profit = 20%

C.P of x ball pens = 100/ 120×96 = Rs. 79.99


= Rs. 80
Cost Price of 90 ball pens= Rs. 200
C.P of 1 Ball pen = Rs. 200 / 90 = Rs. 2.22

Therefore, Rs.80 is the C.P of x ball pens


No. of ball pens bought at Rs. 80 = 80/ 2.22 = 36 (approx)
Thus, 36 ball pens should be sold at Rs. 96 to earn a profit of 20%
Question 16
A man sells an article at a profit of 25%. If he had bought it at a 20% less and sold
it for Rs.36.75 less, he would have gained 30%. Find the cost price of the article
Solution
Let the C.P be the article be Rs. x
Profit = 25%
Original S.P = x + 25 / 100x
= Rs. 1.25x
If he purchased it at 20% less,
C.P = x- 20 / 100x
= Rs. 0.8x
He sold the article at Rs. 36.75 less
So, the selling price = Rs. 1.25x – 36.75
Given, that he would have gained 30% at selling price.
Therefore, gain= S.P-C.P
=( 1.25x -36.75 )– 0.8x
= 0.45x - 36.75
Gain % = 𝒈𝒂𝒊𝒏 × 100
𝑪𝑷
𝟎.𝟒𝟓𝐱 − 𝟑𝟔.𝟕𝟓
So, 30% = 𝟎.𝟖𝐱
𝟎.𝟒𝟓𝐱 − 𝟑𝟔.𝟕𝟓
0.3 = 𝟎.𝟖𝐱

0.3 × 0.8x = 0.45x -36.75


0.24x = 0.45x - 36.75
x = 175
The cost price of the article is Rs. 175.

54
Question 17
A dishonest shopkeeper professes to sell pulses at his cost price but uses a false
weight of 950 gm for each kilogram. Find his gain percentage.
Solution
CP. 0f 1000g = Rs.950
SP 0f 1000g =1000g
he will gain:(1000 - 950 ) gm

Gain%= 𝒈𝒂𝒊𝒏 × 100


𝑪.𝑷
Gain% = 50/ 950×100
𝟓
His gain percentage is 5 % = 5.26%
𝟏𝟗

Question 18

A dealer bought two tables for Rs.3120. he sold one of them at a loss of 15% and
the other at a gain of 36%. Then, he found that each table was sold for the same
price. Find the cost price of each table.
Solution
Given that the selling price is same for both the tables.
Let the C.P of 1 table be x
Then the C.P of the other table be = Rs.3120-x
Loss on the first table = 15%
Therefore, S.P =( 85 / 100) x
= 0.85x
Gain on the second table = 36%
SP of second table = 136 / 100(3120-x)
= 4243.2 -1.36x
Since both the tables have the same S.P

0.85x= 4243.2 -1.36x


2.21x= 4243.2
x= 1920

55
The cost price of the table is Rs.1920
The cost price of the other table is Rs. (3120-1920)
= Rs.1200

Question 19
Mariam bought two fans Rs.3605. she sold one of them at a profit of 15% and the
other one at a loss of 9 %. If Mariam obtained the same amount for each fan, find
the cost price of the each of the fans.
Solution
It is given that the S.P is same for both the fans.
Let the C.P of the first fan be Rs. x
Therefore, C.P of the second fan be Rs. (3605-x)
Profit on the first fan = 15%
Loss on the second fan = 6%

For the first fan, S.P = 115/100 x


= 1.15x
For the second fan, S.P = 91/100 (3605 - x)
= 0.91(3605-x)
= 3280.55- 0.91x
Since, S.P of both the fans is the same

1.15x = 3280.55- 0.91x


2.06x = 3280.55
x = 1592.5
C.P of the first fan = Rs. 1592.5
C.P of the second fan = Rs. (3605-1592.5)
= Rs. 2012.50
The cost prices of the both the fans are Rs.1592.50 and Rs. 2012.50
respectively.
Question 20

56
Some toffees are bought at a rate of 11 for Rs.10 and the same number at the rate
of 9 for Rs.10. if the whole lot is sold at one rupee per toffee, find the gain or loss
percent on the whole transaction.
Solution:
1st condition
CP of 11 toffees = Rs. 10
CP of 1 toffee = Rs. 10/11
2nd condition
CP of 9 toffees = Rs. 10
CP of 1 toffee = Rs. 10/9
Total CP 0f both toffees= 10/11 + 10/9 = 200/99

CP of 1 toffee = 𝟐𝟎𝟎
𝟗𝟗× 𝟐

= 100/99
SP of 1 toffee = Re. 1
Loss = CP - SP
= 100/99-1
=1.01 - 1
= 0.01

Loss% = Loss / CP ×100


= 0.01/ 1.01 × 100
=1%
The total loss on the whole transaction would be 1%
Question 21
A tricycle is sold at a gain of 16%. Had it been sold for Rs.100 more, the gain
would have been 20%. Find the C.P of the tricycle.
Solution
Let the CP of the tricycle be Rs. x
Gain % = 16%

57
Then SP = 116/100 x
= 1.16x
When S.P increases by Rs.100, we get
New SP = 1.16x + 100
Gain % = 20%
Gain % = gain/ CP × 100
𝟏.𝟏𝟔𝐱 +𝟏𝟎𝟎 − 𝐱
20% = 𝒙
0.2 x = 0.16x + 100
X = 2500
The C.P of the cycle is 2500
Thus, C.P of the tricycle is Rs. 2500.
Question 22
Shabana bought 16 dozens ball pens and sold them at a loss of S.P of 8 ball pens.
Find:
(i) Her loss percent
(ii) SP of 1 dozen ball pens, if she purchased these 16 dozens ball pens for Rs.576
Solution
(i) Number of pens bought = 16(12) = 192
Let S.P of one pen be Rs. x
Therefore, S.P of 192 pens = 192x
S.P of 8 pens = Rs. 8x
It is given that S.P of 8 pens is equal to the loss on selling 192 pens.
Therefore, loss= Rs.8x
C.P of 192 pens = Rs 576
Loss = C.P –S.P
8x = 576 – 192x
200x = 579
x= 2.88
Therefore, loss= 8x = RS.23.04
Loss% = 23.04/576 x100
=4%
(ii) SP of 1 pen = Rs.2.88

58
Therefore, S.P of 1 dozen pens = 12x = 12(2.88)
= Rs.34.56

Question 23
The difference between two selling price of a shirt at a profit of 4 % and 5% is
Rs.6. find:
(i) CP of the shirt
(ii) The two selling prices of the shirt
Solution
Let the C.P of both the shirt be RS. X
For 1 shirt profit = 4%
Profit% = Rs. 0.04x
S.P = Rs.1.04x
For 2 shirt profit = 5%
Profit% = Rs. 0.05x
S.P = Rs.1.05x
It is given that the difference between their profits is Rs.6
So, 1.05x-1.04x = 6
= x = Rs.600
Thus, C.P = Rs.600
S.P of 1 shirt 1 = Rs.1.04x = Rs. 1.04 (600) = Rs. 624
S.P of 1 shirt 2 = Rs.1.05x = Rs. 1.05 (600) = Rs. 630
Question 24
Toshiba bought 100 hens for Rs.8000 and sold 20 of these at a gain of 5%. At
what gain percent she must sell the remaining hens so as to gain 20% on the
whole?
Solution
Let SP of 80 hens be x
C.P of 100 hens = RS. 8000
Cost of one hen = 8000/ 100
=Rs. 80
C.P of 20 hens = Rs (80 x 20) = Rs. 1600

59
Gain = 5%

S.P of 20 hens = 105 / 100×1600


= Rs. 1680
C.P of 80 hens = Rs.(80 x 80) = Rs. 6400
Total CP of 100 hens = Rs.8000
Total SP of 100 hens = SP of 20 hens + SP of 80 hens
= 1680 + x
Gain on 100 hens = S.P of 100 hens – C.P of 100 hens
= 1680 + x - 8000
Gain% = (SP - CP) / CP x 100
𝟏𝟔𝟖𝟎 +𝐱 − 𝟖𝟎𝟎𝟎
20%=
𝟖𝟎𝟎𝟎

0.2 x 8000 = x- 6320


1600 +6320 = x
X = 7920
Gain % 80 hens = (7920 - 6400) / 6400×100
= 23.75%
Therefore, Toshiba gained 23.75% on 80 hens.

60
2. DISCOUNT, COMMISSION &
BROKERAGE

⚫ Selling Price: The price at which a product is sold is called the


selling price (SP) of the product.
S.P = 𝟏𝟎𝟎−𝑫% ×M.P.
𝟏𝟎𝟎

⚫ Marked Price:The price on the label of an article/product is


called the marked price or list price. This is the price at which
product is intended to be sold. However, there can be some
discount given on this price and the actual selling price of the
product may be less than the marked price. It is generally
denoted by MP.
M.P = 𝟏𝟎𝟎 ×SP
𝟏𝟎𝟎 − 𝑫%

⚫ Discount:Discount means reducing the price or value of an


object or item. Regular price minus Sale price gives the amount
of discount.

Discount = Marked Price – Selling Price


= M.P. - S.P.

➢ If the discount is given in percent, then the amount of discount


can be found by using the formula,

Discount = Regular Price × Rate of Discount.


M. P.× Discount %

Discounts are offered on the marked price and the selling


price is determined by the discount offered on the marked
price. For the process of simplification, let us assume:

C = Cost price S
= Selling price
M = Market price
D% = Discount%
G% = Gain%

61
Now,
Discount = D% of marked price, M
Discount = Marked Price – Selling Price
Marked Price – Amount of Discount = Selling Price
M (1-D%) = Selling Price
Also, Selling Price = Cost Price + Gain
Thus,
M (1-D%) = C (1 + G%)
Or in other words
Marked Price (1 – Discount%) = Cost Price (1 + Gain%)

⚫ COMMISSION

A producer or manufacturer of goods generally does not sell his goods


directly to the ultimate consumer. There are agents who purchase the
goods from the manufacturer and sell them to the consumer. In a sense,
such agents bring the manufacturers and the consumers together for
transaction. The remuneration which an agent gets for his services in the
transaction is called commission. Most of the business transactions are
made through intermediate persons.

RATE OF COMMISSION

The amount of commission that an agent gets in the transaction depends


on the volume of work done or the services rendered by him. His
commission is based on the value of the goods bought or sold and is
generally fixed on a percentage basis. In some cases, he is paid a
commission on the total sales brought by the agent or on different slabs.
For example, it may be 5% on the first Rs. 10,000/-, 6% on the next Rs.
5,000/- and so on.

Margin: The profit percentage on selling price is known as


MARGIN.

⚫ BROKERAGE :A brokerage provides intermediary services


in various areas, e.g., investing, obtaining a loan, or purchasing
real estate. A broker is an intermediary who connects a seller
and a buyer to facilitate a transaction.Individuals or legal

62
entities can act as brokers. The broker performs its actions
according to the client’s instructions. The broker is then
compensated, receiving either a flat fee or a certain percentage
of the transaction amount.

PRACTICE QUESTIONS

1. Find Selling Price of a toy if Market Price =Rs 650/- and


Discount =10%

Solution:

We have, M.P.=Rs.650 and Discount Rate=10%

Therefore,

Discount in Rs. = 10% of Rs.650


Discount in Rs.=Rs.(10100×650)
=Rs.65

Hence,

S.P.=M.P. - Discount
S.P.= 650 −65
S.P.=585Rs.

2. Find the Market price of a chair, if Shalini paid Rs 3430


after discount of 2%.

Solution:

As shalini paid Rs. 3430/- after discount of 2%. So

S.P.=Rs.3430
Discount_Rate=2%

Therefore,

𝑺.𝑷 × 𝟏𝟎𝟎
M.P =
𝟏𝟎𝟎 − 𝑫%

63
𝟑𝟒𝟑𝟎 × 𝟏𝟎𝟎
M.P.= 𝟏𝟎𝟎− 𝟐
M.P.=Rs.3500

3. Akshay paid Rs. 562.50 for T-shirt in a sale, while


the price tag shows M.P. = Rs 625. Find discount in
percentage.
Solution: We have,

M.P.=625Rs
.S.P.=562.50Rs.

Therefore:

Discount.=M.P.−S.P.
Discount=625−562.50.
Discount= Rs 62.50.

Hence,

Discount rate = discount /MP × 100


Discount rate=62.50/ 625×100%
Discount rate=10%

4. At a clearance sale, all goods are on sale at 45% discount.


If I buy a skirt marked Rs 600, how much would I need to
pay.
Solution: We have,

M.P.=Rs 600
Discount_Rate=45%

Therefore,

S.P.=M.P.(100−Discount%/100)
S.P.=600(100−45/100)
S.P.=600(55/100)
.S.P.=Rs. 330

64
5. After allowing a discount of 12% on the marked price of an
article, it is sold for Rs 880. Find the market price.

Solution : We have,

S.P.=Rs 880
Discount_rate=12%

.Therefore,
M.P = 𝟏𝟎𝟎 ×SP
𝟏𝟎𝟎 − 𝑫%
M.P.= 𝟏𝟎𝟎 × 880
𝟏𝟎𝟎 −𝟏𝟐
.M.P.=Rs 1000

6. A shopkeeper offers his customers 10% discount and still


makes a profit of 26%. What is the actual cost to him of an
article marked Rs 280.
Answer: Shopkeeper makes proit of 26% on selling price item. First, we will
determine the selling price of the item.

Calculation of selling price

We know:

M.P.=Rs280
Discount_Rate=10%
. Therefore,

𝟏𝟎𝟎−𝑫%
S.P.= ×MP
𝟏𝟎𝟎

S.P.=(100- 10) / 100 ×280


S.P.=280(90/100)Rs
.S.P.= Rs.252

Calculation of cost price

Now we that

S.P.=252Rs.
Gain=26%
, Therefore,

65
C.P.= 𝟏𝟎𝟎 ×SP
𝟏𝟎𝟎 + 𝒈𝒂𝒊𝒏%

C.P.=252(100/100+26)Rs
.C.P.=252(100/126)Rs
.C.P.=200Rs.

Hence, the actual cost of the article is Rs 200

7. A dealer purchased a microwave oven for Rs10000. He offered a


discount of 20% on its listed price and still gains 10%. Find the listed
price of the microwave oven.
Solution:

Cost price of the oven = Rs10000, Profit% = 10%.


Thus, selling price = {100 + profit %} /100 × CP
= Rs 100+10)/100×10000
= Rs (110/100)×10000
= Rs 11000.

Now, For the listed price,

M.P = 𝟏𝟎𝟎
×SP
𝟏𝟎𝟎 − 𝑫%

M.P. = 100/(100-20) × 11000


= 100/80 × 11000
= Rs.13750
Hence, the marked price of the microwave oven is Rs 13750

8. Reema purchased following items from a Khadi-Bhandar.


(i) 3 sarees for 560 rupees each.
(ii) 6 bottles of honey for 90 rupees each.
On the purchase, she received a rebate of 12% . How much
total amount did Reema pay?

66
Solution:
Cost of each saree = Rs 560

∴ Cost of 3 sarees = 3 × 560 = Rs 1,680

Cost of each bottle of honey = Rs 90

∴ Cost of 6 bottles of honey = 6 × 90 = Rs 540

Total cost of the items purchased = Rs 1,680 + Rs 540

= Rs 2,220

Total rebate received = 12% of Rs 2,220 = 12/100×2220

= Rs 266.40

∴ Total amount paid by Reema

= Total cost of the items purchased − Total rebate received

= Rs 2,220 − Rs 266.40

= Rs 1,953.60

Thus, the total amount paid by Reema is Rs 1,953.60

9. An article was sold for a net selling price of Rs. 6,225 after
giving a 17% discount on the list price and thus a 24.5%
profit was gained on cost. Find the list price and the cost
price.
Solution:
Given that S.P. = Rs 6,225,
Discount= 17%
profit= 24.5%
MP. = 𝟏𝟎𝟎
𝟏𝟎𝟎−𝑫%
× SP
𝟏𝟎𝟎

= ×6225
𝟏𝟎𝟎− 𝟏𝟕

= .Rs. 7,500
.Also,
𝟏𝟎𝟎
CP = ×SP
𝟏𝟎𝟎+ 𝑷𝒓𝒐𝒇𝒊𝒕 %

67
𝟏𝟎𝟎 × 6225
=
𝟏𝟎𝟎 +
𝟐𝟒.𝟓

= Rs. 5,000
The list price is Rs. 7,500 and the cost price is Rs.5,000.

10. An agent sold Rs. 3,000 worth of articles on 4½ %


commission basis. Find the commission of the agent.

Solution :

The agent sold Rs. 3,000 worth of articles on 4½ % commission


basis Agent‘s commission= 4½ of Rs. 3,000

=Rs( 9 ×3000)
2 ×100

= Rs. 135

11. An agent was paid Rs. 1596 as commission at rate


12% on the sale of bicycles. The selling price of
each bicycle was Rs. 950. Find the number of
bicycles sold by the agents.

Solution:
Price of one bicycle =Rs. 950
commission = 12%
Commission on one bicycle = 12% of 950
= 12/100 ×950
= Rs. 114
No. Of bicycles sold by the agent
= Total commission / Commission on one bicycle
= 1596 / 114
= 14
12. An insurance agent gets commission of 20% on first year
premium, 6% on second and third year‘s premium and 4%
on subsequent years premium on an insurance policy of
Rs. 40,000. Annual rate of premium being Rs. 30 per
thousand. Find the total earning of the agent for which 5
annual premiums have been paid.

68
Solution:
The rate of annual premium is Rs. 30 per thousand.
Annual premium= 30 ×40000 = 1200
1000

Commission for first year = 20


100
× 1200 = 240

Commission for second and third year= 2 ( 6 × 1200)


100
= 2 (72)
= 144
Commission for fourth and fifth year= 2( 4 ×120)
100
=2 (48)
= 96
Total Earnings = 240 + 144 + 96 = 480= Rs. 480

13. At 7% rate of commission, a sales girl, got Rs. 210 on the


sale of combs. Find the value of the sale if the price of
each comb is Rs. 15 per container. Find the number of
containers sold by the sales girl.

Solution:
Since the rate of commission is 7% and she received Rs. 210,
total value of her sales=Rs. = Rs.210 x 100/7
= Rs. 3,000.
The price of each container is Rs. 15.
Total number of containers sold = 3000/15
=200

14. Find the commission on total sales worth Rs. 25,000, if


the rate of commission is 3% on first Rs. 10,000 and 4½ on
sales over Rs. 10,000.
Solution:
Total sales are worth Rs. 25,000.
Commission at 3% on first Rs. 10000=Rs. 10,000× 3%
=Rs. 300
Commission at 4½ % on sales over Rs. 10,000
=Commission at rate 4½% on Rs.15000(25000-10000).
=Rs. 9 × 15,000
2×100
= Rs. 675

69
Total commission = Rs. (300 + 675) = Rs.975

15. A sales man is allowed 5% commission on the total


sales made by him plus a bonus of 1% on the excess of his
sale over Rs. 20,000/-.If the total earning are Rs. 1,450/-on
commission alone, find his total earnings?

Solution:
Let the total sales be Rs. x.
Then commission at 5% on Rs. x=5/100x
=0.05x.
This is given to be Rs. 1450/
0.05x=1450
x=29,000
the amount of total sales is = Rs. 29,000/-
excess of sales over Rs. 20,000= Rs. (29,000-20,000)
= Rs. 9,000/-
Bonus at 1% on Rs. 9,000 =Rs. 𝟏 ×9000 = Rs. 90/-
𝟏𝟎𝟎

16. A mall gave a 10% discount on a watch and still gained


62% profit. How much per cent above cost had the mall
marked the watch for sale?

Solution:
Given that discount%= 10
Profit % = 62.
Let assume C.P. = 100
𝟏𝟎𝟎 + 𝑷𝒓𝒐𝒇𝒊𝒕 %
SP = 𝟏𝟎𝟎
× CP
𝟏𝟎𝟎 + 𝟔𝟐
= 𝟏𝟎𝟎
× 100
= 162
Also,
𝟏𝟎𝟎 − 𝒅𝒊𝒔𝒄𝒐𝒖𝒏𝒕 %
SP = × MP
𝟏𝟎𝟎

MP = 162×𝟏𝟎𝟎
𝟗𝟎

70
MP= Rs. 180

17. What percentage of profit is earned by marking goods 60%


above cost and then giving a 8% discount?

Solution:

Since no price is given,


we assume C.P. = 100
Now L.P. is 60% above C.P
L.P = 160% of C.P.
LP = CP + 60%CP
= 100 + 60% of 100
= 160
Discount = 8% on L.P. = 8% of 160
= 12.80
S.P= L.P. –Discount Amount
= 160 –12.80
= 147.20
Profit= .S.P. –C.P.
= 147.20 –100
= 47.20
Thus, on C.P = 100, the profit is 47.20. Hence the Profit is
47.20%

71
3.PAYROLL
⚫ What Is Payroll?

Payroll is the compensation a business must pay to its employees


for a set period or on a given date. It is usually managed by the
accounting or human resources department of a company. Small-
business payrolls may be handled directly by the owner or an
associate.

‘Payroll’ as a term basically is the list of employees who are


entitled to receive compensation from an organization. Along with
the amount that each employee should receive for the time worked
or tasks performed, payroll refers to a company’s financial record
of payments that are made to the employees by the employer
including wages, salaries, bonuses, incentives, etc.

Payroll can also refer to the list of a company's employees and the
amount of compensation due to each of them. Payroll is a major
expense for most businesses and is almost always deductible,
meaning the expense can be deducted from gross income lowering
the company's taxable income. Payroll can differ from onepay period
to another because of overtime, sick pay, and other variables.

Payroll is the process of paying a company’s employees, which


can include the tracking of hours worked, the calculation of
employee’s pay, and the distribution of payments via
direct deposit directly to their account or by check. Payroll
is a list of employees who get paid by the company.
Payroll also refers to the total amount of money employer
pays to the employees. As a business function, it involves:

1. Developing organization pay policy including flexible


benefits, leave encashment policy, etc.
2. Defining pay slip components like basic, variable pay,
HRA, and LTA
3. Gathering other payroll inputs (e.g., organization’s
food vendor may supply information about the

72
amount to be recovered from the employees for
meals consumed).
4. The actual calculation of gross salary, statutory as
well as non-statutory deductions, and arriving at the
net pay.
5. Releasing employee salary.
6. Depositing dues like TDS, PF, etc. with appropriate
authorities and filing returns.

What is Payroll Processing?

Payroll Processing refers to the complete set of steps


involved in calculating the total remuneration of each
employee. The process typically involves three to four
stages and tasks such as defining salary structures,
gathering employee data, components, deductions,
allowances, and setting up the necessary policies with
respect to taxes and other adjustments, and then
calculating the total salary after adjusting all the company
policies. After the salaries are disbursed, filing, reporting
and providing payslips to employees also comes under
the entire payroll processing cycle.

In simplest words, if payroll is the amount paid by the


employer to employee, payroll processing is the
whole methodology to accurately calculate the net pay
of the employees as per statutory compliances and
company policies.

The formula,
Net Salary = Gross Salary – Gross Deductions
here,

73
Gross Salary = Basic Salary + HRA + All types
of Allowances + Reimbursements + Arrears +
Bonus
Gross Deductions = Professional Tax + Public
Provident Fund + Income Tax + Insurance +
Leave adjustments + Loan repayments (if any)

In depth
In depthexplanation:
explanation:
Salary
Salary==Basic
Basic ++ HRA
HRA+ + Transport
Transport Allowance
Allowance + +
Reimbursements ++Arrears
Reimbursements + Bonus
Arrears + FBP
+ Bonus + FBP
Allowance + Bonus – Professional Tax -Provident
Allowance + Bonus – Professional Tax -Provident
Fund – Income Tax – Insurance-Leave Adjustments
Fund – Income
(If any)- Tax – Insurance-Leave
Loan Repayments (if any)
Adjustments (If any)- Loan Repayments (if any)

What is Salary Slip or Payslip?


A salary slip or pay slip is a document containing a
detailed list about the various components of your salary
along with specific details of employment. It is issued
every month by an employer either in the form of a printed
hard copy or an electronic copy. Ideally a pay slip must
contain a company logo along with company name and
address.

Who gets a Salary Slip?

A salary slip is only available to salaried employees and


your employer is responsible for providing you a copy of
your pay slip every month. Some smaller companies might

74
not regularly provide a salary slip and in such cases, you
can ask your employer for a Salary Certificate. While most
employers provide digital pay slips, other might provide
physical copies of the same.

Format of Salary Slip

Different companies follow different formats of salary slips.


Following is a basic template for a salary slip includes the
following:

• Company name, logo and address, Salary Slip month


and year
• Employee Name, Employee Code, Designation,
Department
• Employee PAN/Aadhaar, Bank Account Number
• EPF Account Number, UAN (Universal Account
Number)
• Total Work Days, Effective Work Days, Number of
Leaves
• Itemised list of Earnings and Deductions
• Gross Pay and Net Pay in numbers as well as words

Salary Slip Sample

The following is a salary slip sample:

75
The above format is just an illustrative example and your
salary slip may include or exclude some of the
components shown in the example.

What are the Components of a Salary Slip?

1. Income/ Earnings

• Basic Salary: This comprises 35% to 40% of your


salary, thus it is perhaps the most important
component of salary. It also serves as a basis for
determining the other components of salary.
• Dearness Allowance (DA): Calculated as a
percentage of your basic pay, DA is given to offset
the impact of inflation. It is completely taxable and
need to be declared while ITR filing. This pay slip
component typically features in case of government
employees.

76
• House Rent Allowance (HRA): It is an allowance to
help people pay their house rent. The amount of HRA
depends on the location and ranges between 40%-50%
of basic pay. You can claim a part of the HRA as a
tax deduction, provided you live in a rented house as
per Section 10 of the Income Tax Act, 1961.
• Performance and Special Allowance: This is given
to encourage employees for a better performance.
This component is completely taxable.
• Other Allowances: This comprises the various
additional allowances paid by an employer for
any reason. An employer may categorize such
allowances under a specific head or group them
as “Other Allowances”.

2. Deductions


• Employees Provident Fund (EPF): This comprises a
compulsory deduction in your salary slip. This
component of your salary slip is at least 12% of your
basic salary and diverted to an EPF account. Notably,
your contribution towards the EPF is exempted from
tax as per Section 80C of Income Tax Act.
• Professional Tax: This pay slip component is levied
on all individuals, including the salaried,
professionals and traders who have an income. It is
levied in some states only and calculated on the
basis of an individual’s tax slab.

• Tax Deductible at Source (TDS): It refers to the


amount of tax deducted by your employer on behalf
of Income Tax department. You can reduce TDS by
investing in tax-saving schemes and submitting the
appropriate documents to your employer.

77
Example:

The income part of the salary slip has a basic salary and
allowances. The same is explained below.

Basic

Basic, as the name suggests, is the basic component of


the salary. It constitutes 35-50% of the salary. It forms the
basis of other components of the salary. At junior levels,
the basic tends to be high. As the employee grows in the
organization, other allowances tend to be higher.
Organizations tend to keep basic low so that the
allowance pay won’t be topped. The salary is 100%
taxable in the hands of the employee. Basic is the first
component on the earnings side of the salary slip.

78
Dearness Allowance

Dearness Allowance paid to offset the impact of inflation


on one’s pay. It is usually 30-40% of the basic pay.
Dearness allowance da is directly based on the cost of
living. Hence it is different for different locations. For
income tax, basic and DA are considered as pay.
Therefore it is taxable. It appears in the earnings side of
the salary slip right after the basic pay.
House Rent Allowance

House Rent Allowance (HRA) is given to employees living


in rented facilities. The HRA depends on the city of
residence of the employee. For a metro city, HRA is 50%
of the basic pay. For all other cities, it is 40% of the basic
pay. Since housing rent allowance is an allowance, it is
exempted from tax up to a specific limit, provided the
employee pays the rent. It appears in the earnings side of
the salary slip. One can save income tax on HRA. The
exemption is the minimum of the following:

• Rent paid annually minus ten percent of the pay


(basic + DA)
• Actual HRA received
• 50% of (basic + dearness allowance da) in case the
location is (Mumbai, Kolkata, Chennai, Delhi) or 40%
of (basic + dearness allowance da) in case of other
cities.

Conveyance Allowance

Conveyance Allowance is the amount an employer pays


an employee to travel to and from work. It is an allowance.
Hence is exempt from tax up to a specific limit. It appears
in the earnings side of the salary slip. One can save
income tax on conveyance allowance. The exemption is
the minimum of the following:

79
• INR 1600 per month
• Actual conveyance allowance received

Medical Allowance

Medical Allowance is the amount an employer pays to an


employee for medical expenses during the term of the
employment. One can save income tax on medical
allowance. However, the employee only receives this
amount on the submission of medical bills as proof. If the
employee fails to submit evidence of medical bills, he/she
will receive the allowance, but it will be fully taxed. In case
the proof is provided, the allowance up to INR 15,000 is
only exempt from tax. It appears in the earnings side of
the salary slip.

Leave Travel Allowance

It’s given by employers to cover the cost of employee


travel while on leave. It includes the travel expenses of the
employee’s immediate family members as well. Proof of
the journey is required to avail deduction subject to certain
limits. Any expenses incurred during the trip apart from
travel do not count towards the leave travel allowance tax
exemption. The exemption is also applicable only for two
journeys undertaken in a block of 4 calendar years. It
appears in the earnings side of the salary slip.

Special Allowance in Salary

Special allowances include performance-based


allowances. These are usually given to encourage
employees to work better. Also, these allowances vary
from company to company. Special allowances are 100%
taxable. It appears in the earnings side of the salary slip.

80
Allowances

Deduction adjusted in salary slip

The deduction part of the salary slip has the professional


tax, TDS and EPF. The same is explained below.

Types of deductions in salary slip

The three types of deductions in a salary slip are

• Professional Tax
• Tax Deducted at Source
• Employee Provident Fund

Professional Tax

Professional tax is a small tax levied by state governments


on earing professionals. It is payable only in a few states.
Namely, Karnataka, West Bengal, Andhra Pradesh,

81
Telangana, Maharashtra, Tamilnadu, Gujarat, Assam,
Chhattisgarh, Kerala, Meghalaya, Orissa, Tripura,
Jharkhand, Bihar, and Madhya Pradesh. It is not only
levied on professionals but to anyone who earns a living
through a medium. This amount is deducted from the
taxable income. Also, it usually amounts to just a few
hundred rupees each month and is subject to the gross
tax slab. It appears on the deductions side of the salary
slip.

Tax deducted at source

It is the amount deducted by the employer on behalf of the


income tax department. It is based on the gross tax slab of
the employee. One can reduce this amount by investing in
tax-exempt investments like equity funds (ELSS), PPF,
NPS, and tax-saving FDs. It appears on the deductions side
of the salary slip. Hence, investing in section 80C.

82
instruments of the Income Tax Act increases your take
home salary. One can invest in mutual funds (ELSS),
submit investment proof to the company and claim TDS
returns.
Employee Provident Fund (EPF)

It is the contribution of the employee to the provident


fund. This qualifies for section 80C of the Income Tax Act.
Provident fund is the accumulation of funds for employee’s
retirement period. The Employees’ Provident Fund
Organisation governs it. 12% of the employee’s basic
salary goes towards EPF. The employer also makes a
similar contribution on behalf of the employees for their
retirement.

However, not all the contributions made for Employee


Provident Fund goes to the provident fund. Out of
employee’s contribution, 8.33% goes to the Employees’

83
Pension Scheme. If their salary is above INR 15,000, the
contribution is INR 1,250. For employees with a salary
below INR 15,000, 8.33% goes towards the Employees’
Pension Scheme. The balance amount is retained in the
EPF scheme. However, employees can opt-out of the EPF
scheme (up to a limit) and invest in better-earning
instruments like equity funds (ELSS). Employee Provident
Fund appears on the deductions side of the salary slip.

Example of Gross Salary and Net Salary

Sumit works as a Marketing manager with ABC Group Ltd.


His gross salary per month is Rs.70,000 while his net-take
home is just Rs. 56,000.

Basic Salary = Rs.25,000

HRA = Rs.20,000

LTA = Rs.10,000

Travel Allowance = Rs.15,000

Total = Rs.70,000

Deductions:

Provident Fund – Rs. 3000

Income Tax - Rs. 1500

Profession Tax - Rs. 500

Loan Deduction - Rs. 9000

Total Deductions - Rs. 14,000

Therefore, Net Salary = Gross Salary – Deductions =


Rs.70,000 – Rs.14,000 = Rs.56,000.

84
Formula of Gross and NET pay

85
Wages can be paid to the employees either on the
basis of time or on the basis of production done by
them.
Thus, there are two system of wage payment:
1. Time rate System.
2. Piece rate System.
1. Time Rate System:
Time rate system is the oldest method of wage
payment. Under this method the employee is paid
on the basis of time worked i.e. a day, a week, a
fortnight or a month irrespective of quantity
produced. It must be remembered here that wages
are paid after the time fixed for work is completed
irrespective of output or completion of the work.
Every worker knows how much wages he will get
after specific period as an arrangement regarding
wages is reached between employees and
management regarding the wage rate.
Formula:
Wages = Number of hours worked × Rate per
hour
For example, A worker is paid at the rate of Rs.10
per hour and he spent 100 hours at work.
So his wages will be:
Wages = Number of hours worked × Rate per hour

86
= 100 × 10
= Rs.1000
2. Piece Rate System:
Under this system the worker is paid on the basis of
output i.e., amount of work done. The earnings of
the workers are governed by the production
quantities and wage rate per piece. The rate of
wages is fixed in advance. Though the time is not
important in this system, it is assumed that the
worker will not take more than the average time to
complete a job. The earnings of the worker depend
upon the speed of his work and his own individual
skill and efficiency.
As against the time rate system where every
employee is paid the same wage, under this system
the wage varies according to the worker. A superior
worker will earn more than the inferior worker. The
higher the output of the worker, the greater are his
wages.
Formula:
Wages = Number of units produced x Rate per
unit
For example, Suppose piece rate prescribed is
Rs.20 per unit produced.
If a worker produces 10 units, his wages will be
as follows:
Wages = Number of units produced x Rate per unit

87
Wages = 10 x 20
Wages = Rs.200

88
UNIT 3:
FINANCIAL MATHEMATICS
(Simple Interest: Meaning,
working examples
Compound Interest: Meaning,
Working examples on Interest
Compounded Continuously,
Compound Amount at changing
rate)

89
➢ Interest is the cost of borrowing money, where the borrower
pays a fee to the lender for the loan.
➢ The interest, typically expressed as a percentage, can be either
simple or compounded.
➢ Simple interest is based on the principal amount of a loan or
deposit. In contrast, compound interest is based on the
principal amount and the interest that accumulates on it in
every period.
➢ Simple interest is calculated only on the principal amount of a
loan or deposit, so it is easier to determine than compound
interest.

⚫ Simple interest:Simple interest is interest calculated on the


principal portion of a loan or the original contribution to
a savings account.

Simple Interest= 𝑷×𝑹×𝑻


𝟏𝟎𝟎
where:
P = Principal amount
R = Annual interest rate
T= Term of loan, in years

Amount = Principal + Interest


A = P + S.I.

A = P + PRT

A = P(1 + RT)

90
⚫ Compound Interest: Money is said to be at interest when the
interest is to be paid to the lender at the end of a year or other
fixed period. In compound interest calculation, the interest is
added to the sum lent and the amount thus obtained becomes
the principal for the next unit of time or the period fixed. This
process is continued until the last period. After a certain period,
the difference between the amount and the principal is called
the compound interest.

C.I. = Principal (1 + Rate%)Time − Principal


C.I. = P((1 + 𝑟 )t - 1)
100

Formula 1: The formula for annual compound interest, including


principal sum, is:

Amount = P(1+ 𝑹 )n
𝟏𝟎𝟎

Example 1:
Find compound interest on Rs. 7500 at 4% per annum for 2 years,
compounded annually.

Solution:

Amount = Rs. [7500 x (1+ 4 )2]


100

91
= Rs. (7500 x 26 x 26)
25 25
= Rs. 8112.

∴ C.I. = Rs. (8112 – 7500) = Rs. 612.

Example 2 :
Find compound interest on Rs. 8000 at 15% per annum for 2 years
4 months, compounded annually.

Solution:

Time = 2 years 4 months = 2 4 years = 21 years


12 3
Total amount = Amount of 2 years compounded annually +
Simple interest of 4 months on Amount of 2 years
compounded annually
1
× 15
2
Amount = Rs. [8000 x (1+15/100) x (1+3 )]
100
= Rs. (8000 x 23/20 x 23/20 x 21/20)

= Rs. 11109.

∴ C.I. = Rs. (11109 – 8000) = Rs. 3109.

Formula 2: When interest is compounded Half-yearly:

Amount = P(1+( 𝑹 ))2n


𝟐 × 𝟏𝟎𝟎

Example1
Find the compound interest on Rs. 10,000 in 2 years at 4% per
annum, the interest being compounded half-yearly.

Solution:

Principal = Rs. 10000; Rate = 2% per half-year;


Time = 2 years = 4 half-years.
Amount = P(1 + ( 𝑹 ))2n
𝟐 × 𝟏𝟎𝟎

∴ Amount = Rs. [10000 x (1+2/100)4]


= Rs. (10000 x 51/50 x 51/50 x 51/50 x 51/50)

= Rs. 10824.32.
92
∴ C.I. = Rs. (10824.32 – 10000) = Rs. 824.32.

Example 2
What is the difference between the compound interests on Rs.
5000 for 11years at 4% per annum compounded yearly and half-
2
yearly?
Solution:
C.I. when interest is compounded half-yearly

C.I. = P((1 + 𝑟 ) 2t - 1)
2 ×100
4 2×3
= 5000( (1 + ) 2 - 1)
200
= Rs. (5000 x(( 51 ) 3 -1 )
50

= Rs. 306.04
C.I. when interest is compounded yearly
Total C.I= C.I. of 1 years compounded annually + Simple
interest of 6 months on Amount of 1 years compounded
annually

P((1 + 𝑟 )t - 𝑃 × 𝑅× 𝑇
= 1) +
100 100
= P((1 + 4 )1 𝑃 × 𝑅× 𝑇
- 1) +
100 100
26 5200 × 4× 1⁄2
= 5000( - 1) +
25 100
= 200 + 104
= Rs.304
∴ Difference = Rs. (306.04 – 304) = Rs. 2.04.

Formula 3: When interest is compounded Quarterly:


93
⁄𝟒
Amount = P(1+ 𝑹 )4n
𝟏𝟎𝟎

Example 1
Find the compound interest on Rs. 16,000 at 20% per annum for 9
months compounded quarterly.

Solution:
Principal = Rs. 16000; Time = 9 months = 3 quarters;

Rate = 20% per annum = 5% per quarter.

∴ Amount = Rs. [16000 x (1+ 𝟓 )3]


𝟏𝟎𝟎

= Rs. (16000 x 21/20 x 21/20 x 21/20)


= Rs. 18522.

∴ C.I. = Rs. (18522 – 16000) = Rs. 2522.

Example 2
Find the compound interest on Rs. 15,625 for 9 months at 16% per
annum compounded quarterly.

Solution:
P = Rs.. 15625, n = 9 months = 3 quarters, R = 16% p.a. = 4%
per quarter.

Amount = Rs.[15625 x (1+ 𝟒 )3]


𝟏𝟎𝟎
= Rs. (15625 x 26/25 x 26/25 x 26/25)
= Rs. 17576.

∴ C.I. = Rs. (17576 -15625) = Rs. 1951.

94
Practice Questions:
Question 1: A sum of Rs 4000 is borrowed and the
rate is 7%. What is the simple and compound interest
for 2 years?
Solution:

Simple Interest = Principle × Rate %× Time

S.I. = 𝑷𝑹𝑻
𝟏𝟎𝟎

⇒ Simple Interest = 4000 × (7 ⁄ 100) × 2

⇒ Simple Interest = 560

∴ The simple Interest for 2 years is Rs. 560

C.I. = P((1 + 𝒓 )t - 1)
𝟏𝟎𝟎
So, Compound Interest = 4000((1 + 7 ) 2 - 1)
100
⇒ Compound Interest = (4000 × 0.1449)

⇒ Compound Interest = 579.6 = Rs.580(approx)

∴ The compound interest for 2 years is Rs. 580

Question 2: A sum of Rs. 25000 becomes Rs. 30000 at


the end of 4 years when calculated at simple interest.
Find the rate of interest.

Solution:
Given,
Principal = P = Rs. 25000
Time = T = 4 years

95
Amount at the end of 4 years = Rs. 30000
SI = Rs. 30000 – Rs. 25000 = Rs. 5000
𝑃𝑅𝑇
SI = 100
𝐒𝐈 × 𝟏𝟎𝟎
⇒R = 𝐏𝐓
𝟓𝟎𝟎𝟎 × 𝟏𝟎𝟎
⇒R=
𝟐𝟓𝟎𝟎𝟎 × 𝟒
⇒ R = 5%
Hence, the rate of interest = 5%

Question 3: Find the compound interest on Rs. 13000


at 10% for 2 years, compounded annually.

Solution:
Given,
Principal = P = Rs. 13000
Rate of interest = r = 10%
Time = t = 2 years
Amount = P(1 + r/100)2
= 13000(1 + 10/100)2
= 13000 (1 + 0.1)2
= 13000(1.1)2
= 13000 × 1.21
= 15730
CI = Amount – Principal
= Rs. 15730 – Rs. 13000
= Rs. 2730
Therefore, the compound interest = Rs. 2730

Questions 4:Find the amount if Rs 20000 is invested at


10% p.a. for 3 years compound annually .
Solution:
Using the formula:A= P [1+ R/100]n
A = 20000 [1 + (10/100)]3
A = Rs. 26620

96
Question 5: Find the CI, if Rs 1000 was invested for 1.5
years at 20% p.a. compounded half yearly.
Solution:
P = Rs. 1000
T = 1.5 years = 3 ( half years)
R = 20%pa = 10%

CI = P [1+(R/100)]n - P
CI = 1000 [1+(10/100)]3- 1000
On Solving, we get
CI = Rs. 331

Question 6: The CI on a sum of Rs 625 in 2 years is Rs


51. Find the rate of interest.
Solution: We know that A = CI + P
A = 625 + 51 = 676
Now going by the formula: A = P [1+(R/100)]n
676 = 625 [1+(R/100)]2
676/625 = [1+(R/100)]2
We can see that 676 is the square of 26 and 625 is the
square of 25
Therefore, (26/25)2 = [1+(R/100)]2
26/25 = [1+(R/100)]
26/25 - 1 = R/100
On solving, R = 4%

Question 7:A sum of money is put on CI for 2 years at


20%. It would fetch Rs 482 more if the interest is payable
half yearly than if it were payable yearly. Find the sum.

Solution: Let the Principal = Rs 100


When compounded annually,
A = 100 [1+ 𝟐𝟎 ]2 = 144
𝟏𝟎𝟎
When compounded half yearly,
A = 100[1+ 𝟏𝟎 ]4 =146.41
𝟏𝟎𝟎

97
Difference, 146.41 - 144 = 2.41
If difference is 2.41, then Principal = Rs 100
𝟏𝟎𝟎
If difference is 482, then Principal = × 482
𝟐.𝟒𝟏

P = Rs 20000.

Question 8: Manish invested a sum of money at CI. It


amounted to Rs 2420 in 2 years and Rs 2662 in 3 years.
Find the rate percent per annum.
Solution:
C.I. of 3rd year
= Amount of 3rd year - Amount of 2nd year
Last year interest = 2662 - 2420
= Rs 242

𝑺.𝑰. × 𝟏𝟎𝟎
Rate % =
𝑷×𝑻

(𝟐𝟒𝟐 × 𝟏𝟎𝟎)
Therefore, Rate% =
(𝟐𝟒𝟐𝟎 × 𝟏)

R% = 10%

Question 9:The difference between SI and CI for 2 years


@ 20% per annum is Rs 8. What is the principal?

Solution: Using the formula:


Difference = P (R/100)2
8 = P[20/100]2
On Solving, P = Rs 200

OR
S.I = 𝑷𝑹𝑻
𝟏𝟎𝟎
𝑷 × 𝟐𝟎 × 𝟐
= = 0.4 P ---------------- (i)
𝟏𝟎𝟎

98
𝑅 t
C.I = P((1 + ) -1 )
100
20 2
= P ((1 + ) - 1)
100
= P (0.44) = 0.44P ------------------- (ii)
According to question ,
C.I - S.I = 8
0.44P - 0.4 P = 8
P = Rs. 200

Important Formula: To find the difference


between SI and CI for 2 years, we use the
formula Difference = P[ 𝐑 ]2
𝟏𝟎𝟎

Question 10:Calculate the Amount and Compound


Interest on Rs. 2000 for 2 years at 10% per year.
Sol:
A = P (1 + R/100)T .
Given,
P = Rs. 2000, R = 10% and T = 2 years
Amount = 2000(1 + 10/100)2.
A = 2000 × (11/10)2
= (2000 X 121)/100
= 2420
So, the Amount = Rs. 2420.
Hence,
Compound Interest = Amount - Principal
= Rs. 2420 – Rs. 2000
= Rs. 420.

99
Question 11:Find the compound interest on Rs. 12,800
for 2 years at per annum.

Sol:Here,

P = Rs. 12,800, R = 25/2% p.a., T = 2 years


Therefore,
2n
A = P(1 + R/200)
= Rs.12800[1 + 25 ]2
200
= Rs.12800 (1 + 25/200)2
= Rs.12800 (1 + 1/8)2
= Rs.12800 [1.125]2
= Rs. 16200
Hence, the Amount = Rs. 16200
Now,
Compound interest = A – P
= Rs. 16200 – Rs. 12800
= Rs. 3400

Question 12:At what rate percent per annum will a sum of


Rs. 10,000 amount to Rs. 14,641 in 4 years compounded
annually?
Sol: Let the required rate be R% per annum
A = 14641, P = Rs. 10000
We know that
A = P (1 + R/100)n
14641 = 10000 (1 + R/100)4
14641/10000 = (1 + R/100)4
(11/10)4 = (1 + R/100)4
11/10 = 1 + R/100
or 11/10 -1 = R/100
(11 -10)/10 = R/100
1/10 = R/100
100/10 = R
10 = R
R = 10% p.a.

100
Question 13 :Calculate the compound interest on Rs.
12000 for 1 years at 10% per annum when compounded
half-yearly.

Sol: Here,

Principal P = Rs. 12000, R = 20% per annum and


n = 2 years.
Therefore,
Amount after 2 years = P (1 + R/200)2n
= Rs.12000 X (1 + 10/200)2x1
= Rs.12000 X (1 + 1/20)2
= Rs.12000 X (21/20)2
= Rs.12000 X 21/20 X 21/20
= Rs.12000 X 441/400
= Rs.13230
Therefore,
Compound interest = Rs. 13230 – Rs. 12000
= Rs. 1230

Question 14:Shyam deposited in a bank Rs. 7500 for 6


months at the rate of 8% p.a. interest compounded
quarterly. Find the amount he received after 6 months.

Sol: Here,

P = Rs. 7500, R = 8% per annum and


n = 6 months = 6/12 = ½ year.
therefore,
Amount after 6 months = P(1 + R/400)4n
= Rs.7500 X (1 + 8/400)4x1/2
= Rs.7500 X (1 + 1/50)2
= Rs.7500 X (51/50)2
= Rs.7803

101
Question 15: In what time will Rs. 2,560,000 amount to
Rs. 2,825,761 at 5% per annum, interest being
compounded half-yearly?

Sol: Here,

Principal P = Rs. 2,560,000,


Amount A = Rs. 2,825,761,

Rate R = 5% per annum


Since, the interest is compounded half-yearly
therefore,
A = P (1 + R/200)2n,

where n is the no. of years


→2,825,761=2,560,000(1 + 5/200)2n
→2,825,761/2,560,000=(41/40)2n
→(41/40)4 =(41/40)2n
→2n=4
→n=4/2 years = 2 years

102
Question 16: A sum of Rs. 220 is to be repaid in two
equal installments. If the rate of interest be 20 %
compounded annually, then what is the value of each
installment?

Sol: Total sum that has to be paid = Rs.220. Rate of


interest = 20%.

For compound interest the principal value of all the


installments are calculated, added
and then equated with the principal value of loan amount.
Amount = P( 1 + 𝑹
) t and
𝟏𝟎𝟎

𝐀𝐦𝐨𝐮𝐧𝐭
P= (𝟏+
𝑹
)𝐭
𝟏𝟎𝟎

Let the value of each instalment =


X
(𝟏 + 𝟐𝟎/𝟏𝟎𝟎)𝟏 (𝟏 + 𝟐𝟎/𝟏𝟎𝟎)𝟐
x = 144.

103
Question 17:If the difference between Simple Interest andCompound
Interest on a certain sum of money in 2 years at 20 % p.a. is Rs. 800,
then find the sum.

Sol:

If difference between Simple Interest & Compound


Interest for 2 years is Rs. 800,
Then
Difference = P[ 𝐑 ]2
𝟏𝟎𝟎
𝟐𝟎 2
800 = P[ ]
𝟏𝟎𝟎
→ P = 20000.
Hence the sum is Rs. 20000.
Question 18:The compound interest on a certain sum of
money for 2 years is Rs. 52 and the simple interest for the
same time at the same rate is Rs. 50. Find the rate %.

Sol:
SI for 2 years = 50,
CI for 2 years = 52.
Therefore
As, SI and CI are same for the first year,
therefore, SI and CI for 1st year = Rs. 25.
So CI for 2nd year = 52 – 25 = 27
Difference in 2nd year’s interest = C.I -S.I
= 27 - 25
=2
Hence,
Rate of interest = 2/25 X 100 = 8%.

104
Question 19:The difference between Compound Interest
and Simple Interest on a certain sum of money at 10 %
per annum for 3 years is Rs. 930. Find the principal if it is
known that the interest is compounded annually.
Sol:
The Simple Interest after three years @ 10% is
30%( 3 × 10%)
The Compound Interest after 3 years @ 10% will
be 1.1 × 1.1 × 1.1 = 1.331

Cumulative rate of Interest is 33.1%


(( 1.331 - 1) × 100%)

Here,
the difference after 3 years is 3.1% and in the question it
is given to be Rs. 930.

Thus, the Principal is 930 × 100/3.1 = Rs. 30000.

105
Question 20 :The population of a town has a constant
growth of 4% p.a. If its present population is 62,500, what
will be its population after two years?
Sol:: Population after 2 years = P (1 + 4/100)2
= 62500 X (104/100)2
= 62500 × 26/25 X 26/25
= 100 × 26 ×26
= 67600.
Hence, the population after two years will be 67,600.

Question 21:The cost of a new washing machine is Rs.


12000. Its value depreciates every year at the rate of 10%.
What will be its value after three years?

Sol:The cost of washing machine = Rs. 12000


Rate of depreciation = 10%, Time = 3 years
Due to depreciation, the price of the washing machine is
reduced.
Therefore,
Price of washing machine after three years
= Rs. 12000 × (1 - 10/100)3
= Rs. 12000 × (90/100)3
= Rs. 12000 × 9/10 x 9/10 x 9/10
= Rs. 8748
Qustion22. An automobile financier claims to be
lending money at simple interest, but he includes the
interest every six months for calculating the principal.

106
If he is charging an interest of 10%, the effective rate
of interest becomes:

Solution:

Let the sum be Rs.100. Then,


S.I. for first 6 months
= Rs. [100 × 10 × 1 / 100 × 2] = Rs.5
S.I. for last 6 months = Rs. [ 𝟏𝟎𝟓 × 𝟏𝟎 × 𝟏 ] = Rs.5.25
𝟏𝟎𝟎 × 𝟐

So, amount at the end of 1 year = Rs. (100 + 5 + 5.25) =


Rs.110.25
So, effective rate = (110.25 ⎯ 100) = 10.25%

Question23. The simple interest on a sum of money in


5 years at 12 % per annum is Rs. 400 less than the
simple interest accrued on the same sum in 7 years at
10 % per annum. Find the sum.

Solution:
Let the sum be P.
→ SI in 5 years at 12 % per annum = P x 12 x 5
100

= 0.6 P
→ SI in 7 years at 10 % per annum = P x 10 x 7
100

= 0.7 P
Now, according to the question,
0.7 P – 0.6 P = 400
→ 0.1 P = 400
→ P = 4000
Thus, the required sum is Rs. 4000

Question24. Aman took a loan from a bank at the rate


of 12% p.a. simple interest. After 3 years he had to pay
Rs. 5400 interest only for the period. The principal
amount borrowed by him was:

107
Solution:
𝟏𝟎𝟎 × 𝟓𝟒𝟎𝟎
Principal = Rs. [ ] = Rs.15000
𝟏𝟐 × 𝟑

Question 25. If Rs. 5000 amounts to Rs. 5832 in two


years compounded annually, find the rate of interest
per annum.

Solution:

Here, P = 5000, A = 5832, n = 2

Amount = P(1+ 𝑹 )n
𝟏𝟎𝟎

→ 5832 = 5000 [1 + (R / 100)]2


→ [1 + (R / 100)]2 = 5832 / 5000
→ [1 + (R / 100)]2 = 11664 / 10000
→ [1 + (R / 100)] = 108 / 100
→ R / 100 = 8 / 100
→R=8%
Thus, the required rate of interest per annum in 8 %

Question 26. Find the compound interest on Rs.5000 for


1 year at 8% per annum, compounded half-yearly.

Solution:

Rate of interest = 8% per annum

= 4% per half-year.

Time = 1 year = 2 half-years

Original principal = Rs.5000.

Interest for the first half-year = Rs.5000×4×1


100

108
= Rs.200.

Amount at the end of the first half-year = Rs.(5000 + 200)


= Rs.5200.

Principal for the second half-year = Rs.5200.

Interest for the second half-year = Rs.5200×4×1


100

= Rs.208.

Amount at the end of the second half-year = Rs.(5200 +


208) = Rs.5408.

Therefore, compound interest = Rs.(5408 - 5000)

= Rs.408.

Question 26. Find the compound interest on Rs.10000 for


1 year at 10% per annum, compounded half-yearly.

Solution:

Rate of interest = 10% per annum

= 5% per half-year.

Time = 1 year = 2 half-years

Original principal = Rs.10000.

Interest for the first half-year = Rs.10000×5×1


100

= Rs.500.

Amount at the end of the first half-year = Rs.(10000 + 500)


= Rs.10500.

109
Principal for the second half-year = Rs.10500.

Interest for the second half-year = Rs.10500×5×1


100

= Rs.525.

Amount at the end of the second half-year = Rs.(10500 +


525) = Rs.11025.

Therefore, compound interest = Rs.(11025 - 10000)

= Rs.1025.

Question27. Calculate the compound interest for the


second and third year on Rs. 20,000 invested for 4 years
at 10% p.a.

Solution:
Interest for the first year = 𝑃×𝑅×𝑇
100

20000×1×10
= 100
= Rs. 2,000

Amount at the end of first year or principal for second year


= Rs. (20,000 + 2,000) = Rs. 22,000

22000×1×10
Interest for second year = = Rs. 2,200
100

Amount at the end of second year or principal for 3rd year

= Rs. (22,000 + 2,200)

= Rs. 24,200

110
24200×1×10
Interest for third year =
100

= Rs. 2,420

Therefore compound interest for the second and third year


on Rs. 20,000 invested for 4 years at 10% p.a. are Rs.
2,200 and Rs. 2,420 respectively.

Question 28. Vijay borrowed Rs. 5,000 and agreed to pay


interest at the rate of 10% , 12%, and 14% for the first,
second and third year respectively. Find the total amount
he had to pay after 3 years.
Solution:
Original principal = Rs.5,000.

Rate of interest = 10%, 12%, and 14% for the first, second
and third year respectively.
𝑹
A= P (𝟏 + )𝒏
𝟏𝟎𝟎
𝟏𝟎 𝟏𝟐 𝟏𝟒

= 5000(𝟏 + 𝟏𝟎𝟎)(𝟏 + 𝟏𝟎𝟎)(𝟏 + 𝟏𝟎𝟎 )

=5000 × 11/10 ×28/25 ×57/50


=Rs. 7,022.40
Question 29. Find the compound interest on Rs.5000 for
3 years at 8% per annum, compounded annually.

Solution:

111
𝑹
A= P (𝟏 + )𝒏
𝟏𝟎𝟎
𝟖
= 5000 (𝟏 + )𝟑
𝟏𝟎𝟎
=6298.56

Therefore, compound interest = Rs. (6298.56 - 5000) =


Rs.1298.56

Question 30. Find the compound interest on Rs.25000 for


3 years at 6% per annum, compounded annually.

Solution:
𝑹
A= P (𝟏 + )𝒏
𝟏𝟎𝟎
𝟔
= 25000 (𝟏 + )𝟑
𝟏𝟎𝟎
= 29775.4
Therefore, compound interest = Rs.(29775.40 - 25000) =
Rs.4775.40.

Question 31. The simple interest on a sum of money


for 3 years at 6²/₃ % per annum is Rs. 6750. What will
be the compound interest on the same sum at the
same rate for the same period, compounded annually?

Solution:

Given, SI = Rs. 6750, R = 20 % p.a. and T = 3 years.


3

𝟏𝟎𝟎 × 𝐒𝐈
sum =
𝐑×𝐓

= Rs. (100 × 6750 × ³/₂₀ × 1/3 ) = Rs. 33750.

112
20
Now, P = Rs. 33750, R = % p.a. and T = 3 years.
3

Therefore, amount after 3 years

20
= Rs.{33750 × (1 + ( )}³ [using A = P (1 + R/100)ᵀ]
3 × 100

= Rs. (33750 × 16/15 × 16/15 × 16/15) = Rs. 40960.

Thus, amount = Rs. 40960.

Hence, compound interest = Rs. (40960 - 33750)

= Rs. 7210.

Question 32. The difference between the compound


interest, compounded annually and the simple interest
on a certain sum for 2 years at 6% per annum is Rs. 18.
Find the sum.
Solution:
Let the sum be Rs. 100. Then,

SI = Rs. (100 × 6 × 2) = Rs.12


100

and compound interest

= Rs. {100 × (1 + 6/100)² - 100}

= Rs. {(100 × 53/50 × 53/50) - 100}

= Rs. (2809 - 100)


25

= Rs. 309/25

Therefore, (CI) - (SI) = Rs. (309/25 – 100)

113
= Rs. 9/25

If the difference between the CI and SI is Rs. 9/25, then


the sum = Rs. 100.

If the difference between the CI and SI is Rs. 18, then the


sum = Rs. (100 × 25/9 × 18 )

= Rs. 5000.

Hence, the required sum is Rs. 5000.

Alternative method

Let the sum be Rs. P.

Then, SI = Rs. (P × 6/100 × 2) = Rs. 3P/25

And, CI = Rs. {P × (1 + 6/100)² - P}

= Rs. {(P × 53/50 × 53/50) - P}

= Rs. (28092500P - P)

= Rs. (309P/2500)

(CI) - (SI) = Rs. (309P/2500 – 3P/25)

= Rs. (9P/2500)

Therefore, 9P/2500 = 18

- P = 2500 × 18/9

- P = 5000.

Hence, the required sum is Rs.5000.

114
Question 33. A certain sum amounts to Rs. 72900 in 2
years at 8% per annum compound interest,
compounded annually. Find the sum.

Solution:

Let the sum be Rs. 100. Then,

amount = Rs. {100 × (1 + 8/100)²}

= Rs. (100 × 27/25 × 27/25) = Rs. (2916/25)

If the amount is Rs. 2916/25 then the sum = Rs. 100.

If the amount is Rs. 72900 then the sum = Rs. (100 ×


25/2916 × 72900) = Rs. 62500.

Hence, the required sum is Rs. 62500.

Alternative method

Let the sum be Rs. P. Then,

amount = Rs. {P × (1 + 8/100)²}

= Rs. {P × 27/25 × 27/25} = Rs. (729P/625)

Therefore, 729P/625 = 72900

- P = (72900 × 625)/729

- P = 62500.

Hence, the required sum is Rs. 62500.

Question 34. At what rate per cent per annum will Ron
lends a sum of Rs.2000 to Ben. Ben returned after 2
years Rs.2205, compounded annually?

115
Solution:

Let the required rate be R% per annum.

Here, A = Rs. 2205, P = $Rs.2000 and n = 2 years.

Using the formula A = P(1 + R/100)ⁿ,

2205 = 2000 × ( 1 + R/100)²

⇒ (1 + R/100)² = 2205/2000 = 441/400 = (21/20)²

⇒ ( 1 + R/100) = 21/20

⇒ R/100 = (21/20 – 1) = 1/20

⇒ R = (100 × 1/20) = 5

Hence, the required rate of interest is 5% per annum.

Question 35. A man deposited Rs.1000 in a bank. In


return he got Rs.1331. Bank gave interest 10% per
annum. How long did he kept the money in the bank?

Solution:

Let the required time be n years. Then,

amount = Rs. {1000 × (1 + 10/100)ⁿ}

= Rs. {1000 × (11/10)ⁿ}

Therefore, 1000 × (11/10)ⁿ = 1331 [since, amount = Rs.


1331 (given)]

⇒ (11/10)ⁿ = 1331/1000

= 11 × 11 × 11/ 10 × 10 × 10

116
= (11/10)³

⇒ (11/10)ⁿ = (11/10)³

⇒ n = 3.

Thus, n = 3.

Hence, the required time is 3 years.

Question 36. Ron borrows Rs.10,000 at a compound


interest rate of 8% per annum. If he repays Rs. 2000 at
the end of each year, find the sum outstanding at the end
of the third year.

Solution:

For the first year:

Principal = Rs. 10,000

Rate = 8 %

Time = 1 year

Therefore, interest = Rs .𝑃×𝑅×𝑇


100

10000×8×1
= Rs.
100

= Rs.80000/100

= Rs. 800

Therefore, the amount of loan after 1 year

= Principal + Interest

= Rs. 10,000 + Rs. 800

= Rs. 10,800

117
Ron pays back Rs. 2,000 at the end of the first year.

So, the new principal at the beginning of the second year

= Rs. 10,800 -Rs.2,000 = Rs. 8,800

Therefore, for the second year:

Principal = Rs. 8,800

Rate = 8 %

Time = 1 year

Therefore, interest = Rs. 𝑃×𝑅×𝑇


100

8,800×8×1
= Rs.
100

= Rs.70400/100

= Rs. 704

Therefore, the amount of loan after 2 year = Principal +


Interest = Rs. 8,800 + Rs. 704

= Rs. 9504

Ron pays back Rs. 2,000 at the end of the second year.

So, the new principal at the beginning of the third year =


Rs. 9504 - Rs. 2,000
= Rs. 7504

Therefore, for the third year:

Principal = Rs. 7504

Rate = 8 %

Time = 1 year

118
Therefore, interest = Rs. 𝑃×𝑅×𝑇
100

7504×8×1
= Rs.
100

= Rs.60032/100

= Rs. 600.32

Therefore, the amount of loan (outstanding sum) after 3


year

=Principal+Interest = Rs.
7504 + Rs. 600.32

= Rs.
8104.32

Question 38. Davis invests Rs. 20,000 at the beginning of


every year in a bank and earns 10 % annual interest,
compounded at the end of the year. What will be his
balance in the bank at the end of three years.
Solution:

For the first year:

Principal = Rs. 20,000

Rate = 10 %

Time = 1 year

Therefore, interest = Rs. 𝑃×𝑅×𝑇


100

20000×10×1
= Rs.
100

= Rs.200000/100

119
= Rs. 2000

Therefore, the amount at the end of the 1 year = Principal


+ Interest

= Rs. 20,000 + Rs. 2000

= Rs. 22,000

Davis deposits Rs. 20,000 at the beginning of the second


year.

So, the new principal for the second year = Rs. 22,000 +
Rs. 20,000

= Rs. 42,000

Therefore, for the second year:

Principal = Rs. 42,000

Rate = 10 %

Time = 1 year

Therefore, interest = Rs .𝑃×𝑅×𝑇


100

42000×10×1
= Rs.
100

= Rs.420000/100

= Rs.4,200

Therefore, the amount at the end of the 2 year = Principal


+ Interest

= Rs. 42,000 + Rs. 4,200

= Rs. 46,200

120
Davis deposits Rs. 20,000 at the beginning of the third
year.

So, the new principal for the third year = Rs. 46,200 +
Rs.20,000 = Rs. 66,200

Therefore, for the third year:

Principal = Rs. 66,200

Rate = 10 %
Time = 1 year

Therefore, interest = Rs. 𝑃×𝑅×𝑇


100

66200×10×1
= Rs.
100

= Rs. 662000/100

= Rs. 6620

Therefore, the amount at the end of the 3 year = Principal


+ Interest

= Rs. 66,200 + Rs. 6,620

= Rs. 72,820

Therefore, the balance in the bank at the end of thee


years will be Rs. 72,820.

2.2 Continuous Compound Interest


➢ Continuously compounded interest is interest that is
computed on the initial principal, as well as all interest
other interest earned. The idea is that the principal will
receive interest at all points in time, rather than in a
discrete way at certain points in time.

121
➢ The continuous compound interest formula is used to
determine the interest earned on an account that is
constantly compounded, necessarily leading to an
infinite amount of compounding periods.
➢ The effect of compounding is earning interest on
investment, or at times paying interest on a debt
that is reinvested to earn additional money that
would not have been gained based on the principal
balance alone.
➢ By earning interest on prior interest, one can earn at an
exponential rate. The continuous compounding formula
takes this effect of compounding to the furthest limit.
Instead of compounding interest on a monthly,
quarterly, or annual basis, continuous compounding
will efficiently reinvest gains perpetually.
➢ Continuously compounded interest assumes
interest is compounded and added back into the
balance an infinite number of times.
➢ Continuous compounding is used to show how much a
balance can earn when interest is constantly accruing.
For investors, they can calculate how much they expect
to receive from an investment earning a continuously
compounding rate of interest.

Formula for Continuous Compound


Interest

A = P × ert

Where,

• A = Amount of money after a certain amount of time


• P = Principle or the amount of money you start with

122
• e = Napier’s number, which is approximately 2.7183
• r = Interest rate and is always represented as a
decimal
• t = Amount of time in years

Practice Questions :
Question 1: An amount of Rs. 2340.00 is deposited in a
bank paying an annual interest rate of 3.1%, compounded
continuously. Find the balance after 3 years.

Solution:

Use the continuous compound interest formula,

A = P × ert

123
Given P = 2340

r = (3.1 / 100) = 0.031

t=3

Here: e stands for the Napier’s number, which is


approximately 2.7183.

However, one does not have to plug this value in the


formula, as the calculator has a built-in key for e.
Therefore,

A = 2340 e0.031(3) ≈ 2568.06


So, the balance after 3 years is approximately
Rs. 2,568.06.

Question 2: Tina invested Rs.3000 in a bank that pays an


annual interest rate of 7% compounded continuously.
What is the amount she can get after 5 years from the
bank?

Solution:

To find: The amount after 5 years.

The initial amount is P = Rs.3000.

The interest rate is, r = 7% = 7/100 = 0.07.

Time is, t = 5 years.

Substitute these values in the continuous compounding


formula,

A = Pert

124
A = 3000 × e0.07(5) ≈ 4257
The amount after 5 years = Rs.4,257.

Question 3: What should be the rate of interest for the


amount of Rs.5,300 to become double in 8 years if the
amount is compounding continuously?

Solution:

To find: The rate of interest, r.

The initial amount is, P = Rs.5,300.

The final amount is, A = 2(5300) = Rs.10,600.

Time is, t = 8 years.

Substitute all these values in the continuous compound


interest formula,

A = Pert
10600 = 5300 × er (8)
Dividing both sides by 5300,
2 = e8r
Taking "ln" on both sides,
ln 2 = 8r
Dividing both sides by 8,
r = (ln 2) / 8 ≈ 0.087 (using calculator)
So the rate of interest = 0.087 × 100 = 8.7
The rate of interest = 8.7%.

125
Question 4: Jim invested Rs.5000 in a bank that pays an
annual interest rate of 9% compounded continuously.
What is the amount he can get after 15 years from the
bank?

Solution:

To find: The amount after 15 years.

The initial amount is P = Rs.5000.

The interest rate is, r = 9% = 9/100 = 0.09.

Time is, t = 15 years.

Substitute these values in the continuous compounding


formula,

A = Pert
A = 5000 × e0.09(15) ≈ 19287
The amount after 15 years = Rs.19,287.

2.3 Compound Interest at


Changing Rate :

When the rate of compound interests for


successive/consecutive years are different
(r 1%, r 2%, r 3%, r 4%, .................. ) then:
A = P( 1 + 𝑟1 )(1 + 𝑟2 )(1 + 𝑟3 ) .............
100 100 100

Where,

A = amount;

126
P = principal;
r 1, r 2, r 3, r 4 .......... = rates for successive years.

Practice Questions:
1. If the rate of compound interest for the first,
second and third year be 8%, 10% and 15%
respectively, find the amount and the compound
interest on Rs. 12,000 in 3 years.
Solution:
The man will receive an interest of 8% in the first
year, 10% in the second year and 15% in the third
year.
Therefore,
A = P( 1 + 𝒓𝟏 )(1 + 𝒓𝟐 )(1 + 𝒓𝟑 )
𝟏𝟎𝟎 𝟏𝟎𝟎 𝟏𝟎𝟎

⟹ A = Rs. 12,000(1 + 8/100)(1 + 10/100)


(1+15/100)
⟹ A = Rs. 12,000 (1 + 8/100)(1 + 10/100)(1 +
15/100)
⟹ A = Rs. 12,000 × 27/25 × 11/10 × 23/20
⟹ A = Rs.12,000 × 6831/5000
⟹ A = Rs. 16,394.40
Therefore,
127
the required amount = Rs. 16,394.40
Therefore, the compound interest = Final amount -
Initial principal
C.I = Rs. 16,394.40 - Rs. 12,000
= Rs. 4,394.40
2. Find the compound interest accrued by Aaron
from a bank on Rs.16000 in 3 years, when the rates
of interest for successive years are 10%, 12% and
15% respectively.
Solution:
For the first year:
Principal = Rs. 16,000;
Rate of interest = 10% and
Time = 1 years.

Therefore, interest for the first year = 𝑃×𝑅×𝑇


100
16000×10×1
=Rs.
100

= Rs.160000/100
= Rs. 1,600
Therefore,
the amount after 1 year = Principal + Interest
= Rs.16,000 +Rs. 1,600

128
= Rs. 17,600
For the second year,
the new principal is Rs. 17,600
Rate of interest = 12% and
Time = 1 years.
Therefore,

the interest for the second year =𝑃×𝑅×𝑇


100
17600×12×1
= Rs.
100

= Rs.211200/100
= Rs. 2,112
Therefore,
the amount after 2 year = Principal + Interest
= Rs. 17,600 + Rs. 2,112
= Rs. 19,712
For the third year,
the new principal is Rs. 19,712
Rate of interest = 15% and
Time = 1 years.
Therefore,

129
the interest for the third year = 𝑃×𝑅×𝑇
100
19712×15×1
= Rs.
100

= Rs. 295680/100
= Rs.2,956.80
Therefore,
the amount after 3 year = Principal + Interest
= Rs. 19,712 + Rs. 2,956.80
= Rs. 22,668.80
Therefore,
the compound interest accrued = Final amount -
Initial principal
= Rs.22,668.80 - Rs. 16,000
= Rs 6,668.80
3. A company offers the following growing rates of
compound interest annually to the investors on
successive years of investment.
4%, 5% and 6%
(i) A man invests Rs. 31,250 for 2 years. What
amount will he receive after 2 years?
(ii) A man invests Rs. 25,000 for 3 years. What will
be his gain?

130
Solution:
The man will get 4% for the first year, which will be
compounded at the end of the first year. Again for
the second year, he will get 5%. So,
A = P( 1 + 𝒓𝟏 )(1 + 𝒓𝟐 )
𝟏𝟎𝟎 𝟏𝟎𝟎

⟹ A = Rs. 31250(1 + 4/100)(1 + 5/100)


⟹ A = Rs. 31250 × 26/25 × 21/20
⟹ A = Rs. 34,125
Therefore, at the end of 2 years he will receive Rs.
34125.
(ii) The man will receive an interest of 4% in the first
year, 5% in the second year and 6% in the third
year.
Therefore,
A = P( 1 + 𝒓𝟏 )(1 + 𝒓𝟐 )(1 + 𝒓𝟑 )
𝟏𝟎𝟎 𝟏𝟎𝟎 𝟏𝟎𝟎

⟹ A = Rs. 25000(1 + 4/100)(1 + 5/100)(1 + 6/100)


⟹ A = Rs.25000 × 26/25 × 21/20 × 53/50
⟹ A = Rs. 28,938
Therefore, he gain = Final amount - Initial principal
= Rs. 28,938 - Rs. 25000
= Rs. 3,938

131
4. The population of a village increases by 10%
every year. If the present population is 6000, what
will be the population of the village after 3 years?
Solution:
The present population P = 6000,
Rate (r) = 10
Unit of time being year (n) = 3
𝒓 n
Q = P(1 + ) , where Q is quantity in future
𝟏𝟎𝟎
⟹ Q = 6000(1 + 10/100)3
⟹ Q = 6000(1 + 1/10)3
⟹ Q = 6000(11/10)3
⟹ Q = 6000 × (11/10) × (11/10) × (11/10)
⟹ Q = 7986
Therefore, the population of the village will be 7986
after 3 years.
5. The present population of Berlin is 20,00,000. If
the rate of increase of population of Berlin at the
end of a year is 2% of the population at the
beginning of the year, find the population of Berlin
after 3 years?
Solution:
Population of Berlin after 3 years

132
Q = P(1 + 𝒓 )n
𝟏𝟎𝟎
⟹ Q = 200000(1 + 2/100)3
⟹ Q= 200000(1 + 1/50)3
⟹ Q= 200000(51/50)3
⟹ Q= 200000(51/50) × (51/50) × (51/50)
⟹ Q = 21,22,416
Therefore, the population of Berlin after 3 years =
21,22,416
6. A man buys a plot of land for Rs. 150000. If the
value of the land appreciates by 12% every year
then find the profit that the man will make by selling
the plot after 2 years.
Solution:
The present price of the land, P = Rs. 150000, r =
12 and n = 2

Q = P(1 + 𝒓 )n
𝟏𝟎𝟎
⟹ Q = Rs. 150000(1 + 12/100)2

⟹ Q = Rs. 150000(1 + 3/25)2


⟹ Q = Rs. 150000(28/25)2
⟹ Q = Rs. 150000 × (28/25) × (28/25)

133
⟹ Q = Rs.188160
Therefore, the required profit = Q – P
= Rs. 188160 - Rs. 150000
= Rs. 38160
7. The present population of a town is 75,000. The
population increases by 10 percent in the first year
and decreases by 10% in the second year. Find the
population after 2 years.
Solution:
Here, initial population P = 75,000, population
increase for the first year = r1% = 10% and
decrease for the second year = r2% = 10%.
Population after 2 years:

Q = P(1 + 𝒓 )n × ( 1 - 𝒓 )n
𝟏𝟎𝟎 𝟏𝟎𝟎
⟹ Q = 75,000(1 + 10/100)(1 - 10/100)
⟹ Q = 75,000(1 + 1/10)(1 - 1/10)
⟹ Q = 75,000(11/10)(9/10)
⟹ Q = 74,250
Therefore, the population after 2 years = 74,250

134
8. A man starts a business with a capital of
Rs.10,00,000. He incurs a loss of 4% during the first
year. But he makes a profit of 5% during the second
year on his remaining investment. Finally, he makes
a profit of 10% on his new capital during the third
year. Find his total profit at the end of three years.
Solution:
Here, initial capital P = 1000000, loss for the first
year = r1% = 4%, gain for the second year = r2% = 5%
and gain for the third year = r3% =10%
Q = P( 1 + 𝒓𝟏 )(1 + 𝒓𝟐 )(1 + 𝒓𝟑 )
𝟏𝟎𝟎 𝟏𝟎𝟎 𝟏𝟎𝟎

⟹ Q = Rs.1000000(1 - 4/100)(1 + 5/100)(1 +


10/100)
Therefore, Q = Rs.1000000 × 24/25× 21/20 × 11/10
⟹ Q = Rs.200 × 24 × 21 × 11
⟹ Q = Rs.11,08,800
Therefore, profit at the end of three years =
Rs.11,08,800 - Rs.10,00,000
= Rs.108800
9. The price of a machine depreciates by 10% every
year. If the machine is bought for Rs. 18000 and
sold after 3 years, what price will it fetch?
Solution:

135
The present price of the machine, P = Rs. 18000, r
= 10, n = 3
Q = P(1 - 𝒓 )n
𝟏𝟎𝟎

⟹ Q = 18000(1 - 10/100)3
⟹ Q = 18000(1 - 1/10)3
⟹ Q = 18000(9/10)3
⟹ Q = 18000 × (9/10) × (9/10) × (9/10)
9×9×9
⟹ Q = 18000 × ( )
10×10×10

⟹ Q = 18 × 81 × 9
= 13122
Therefore, the machine will fetch 13,122 after 3
years.

10. The value of a machine in a factory depreciates


at 10% of its value at the beginning of the year. If its
present value be Rs. 60,000, what will be its
estimated value after 3 years?
Solution:
Let the present value of the machine (P) = Rs.
10000, r = 10, n = 3
Q = P(1 - 𝒓 )n
𝟏𝟎𝟎

136
⟹ Q = 60,000(1 - 10/100)3
⟹ Q = 60,000(1 - 1/10)3
⟹ Q = 60,000(9/10)3
⟹ Q = 60,000 × (9/10) × (9/10) × (9/10)
9×9×9
⟹ Q = 60,000 × ( )
10×10×10

⟹ Q = 43,740
Therefore, the value of the machine will be
Rs.43,740 after 3 years.
11. The price of a car depreciates by 20% every
year. By what percent will the price of the car
reduce after 3 years?
Solution:
Let the present price of the car be P. Here, r = 20
and n = 3
Q = P(1 - 𝒓 )n
𝟏𝟎𝟎

⟹ Q = P(1 - 20/100)3
⟹ Q = P(1 - 1/5)3
⟹ Q = P(4/5)3
⟹ Q = P × (4/5) × (4/5) × (4/5)

⟹ Q = (64𝑃)
125

137
Therefore, the reduced price =(64𝑃); so reduction in
125
price = P - (64𝑃) = (61𝑃)
125 125

Therefore, the percent reduction in price = ( 61𝑃 )) ×


125 P
100% = 61/125× 100% = 48.8%

12. The cost of a school bus depreciates by 10%


every year. If its present worth is Rs. 18,000; what
will be its value after three years?
Solution:
The present population P = 18,000,
Rate (r) = 10
Unit of time being year (n) = 3
Now applying the formula of depreciation we get:
Q = P(1 - 𝒓 )n
𝟏𝟎𝟎

⟹ Q = Rs.18,000(1 - 10/100)3
⟹ Q = Rs.18,000(1 - 1/10)3
⟹ Q = Rs.18,000(9/10)3
⟹ Q = Rs.18,000 × (9/10) × (9/10) × (9/10)
⟹ Q = Rs.18 × 81 × 9
= Rs.13,122

138
Therefore, the value of the school bus will be Rs.
13,122 after 3 years.

139
UNIT 4:
MATRICES AND DETERMINANTS &
SIMULTANEOUS LINEAR EQUATIONS
( Matrix: Definition of a Matrix,
Matrix operations, Working
examples
Determinants: : Definition,
Properties of determinants.
Applications in Business
Problem,
Solution of Simultaneous
equations, Working examples )

140
INTRODUCTION:
When some numbers are arranged in rows and
columns and are surrounded on both sides by
square brackets, we call it as a matrix. Matrix or
matrices have very important applications in
mathematics.
Matrix refers to an ordered rectangular
arrangement of numbers which are either real or
complex or functions. We enclose Matrix by [ ] or (
).
Many scientific fields use Matrices in some form or
the other. You will find it in physics like
electromagnetism, optics, quantum mechanics and
more. Further, it is present in computer graphics,
like probability, page rank algorithm and more.
Finally, matrix calculus is also beneficial. It helps in
generalizing classical analytical notions like
derivatives and exponentials to high dimensions.
Moreover, graphics software make use of it while
processing linear transformations in order torender
images.
Definition of a Matrix
Matrix is an ordered rectangular arrangement of
numbers (real or complex) or functions which may
be represented as

141
Matrix is enclosed by [ ] or ( )
What is a Matrix?
Suppose we wish to express the information that
Ram has 20 pens. We may express it as [20] with
the understanding that the number inside [ ] is the
number of pens that Ram has. Now, if we have to
express that Ram has 20 pens and 7 pencils. We
may express it as
[20 7] with the understanding that first number
inside [ ] is the number of pens while the other one
is the number of pencils.
Let us now suppose that we wish to express the
information of possession of pens and pencils by
Ram and his two friends Rohan and Yash which is
as follows:
Ram has 20 pens and 7 pencils,
Rohan has 15 pens and 5 pencils,
Yash has 12 pens and 3 pencils.

Now, this could be arranged in tabular form as follows,

Pens Pencils

Ram 20 7

142
Rohan 15 5

Yash 12 3

and this can be expressed as,

20 7
[15 5]
12 3

In the above arrangement, the entries in the first


column represent the number of pens possessed by
Ram, Rohan, and Yash, respectively and the
entries in the second column represents the number
of pencils possessed by Ram, Rohan, and Yash,
respectively.
Order of a Matrix: If a matrix has m rows and n
columns, then its order is written as m × n. If a
matrix has order m × n, then it has mn elements.

143
Types of Matrices
1) Row Matrix
A row matrix has only one row but any number of
columns. A matrix is said to be a row matrix if it has
only one row. For example,

A=[−𝟏/𝟐 √𝟓 𝟐𝟑]

is a row matrix of order 1 × 3. In general,


A = [aij]1 × n is a row matrix of order 1 × n.
2) Column Matrix
A column matrix has only one column but any
number of rows. A matrix is said to be a column
matrix if it has only one column. For example,
𝟎
B= [ 𝟑 ]
−𝟏

is a column matrix of order 3 × 1. In general,


B = [bij]m × 1 is a column matrix of order m × 1.
3) Square Matrix
A square matrix has the number of columns equal
to the number of rows. A matrix in which the
number of rows is equal to the number of
columns is said to be a square matrix. Thus an m ×
n matrix is said to be a square matrix if

144
m = n and is known as a square matrix of order ‘n’.
For example,
3 −1 0
A= [5 4 1 ]
8 2 −3

is a square matrix of order 3. In general,


A = [aij] m × m is a square matrix of order m.
4) Rectangular Matrix
A matrix is said to be a rectangular matrix if the
number of rows is not equal to the number of
columns. For example,
6 0 4
A= [ ]
7 −10 6

is a matrix of the order 2 × 3


5) Diagonal matrix
A square matrix whose all the elements except the
diagonal elements are zeroes, is called a diagonal
matrix,
3 0 0
e.g. A= [0 −3 0 ]
0 0 −8
In general, A = [aij]m×m is a diagonal matrix, if aij = 0,
when i ≠ j.
6) Scalar Matrix

145
A diagonal matrix whose all diagonal elements are
same (non-zero), is called a scalar matrix,
𝟐 𝟎 𝟎
e.g. A= [𝟎 𝟐 𝟎]
𝟎 𝟎 𝟐

In general, A = [aij]m×n is a scalar matrix,


if aij = 0, when i ≠ j, aij = k (constant), when i = j.
Note: A scalar matrix is a diagonal matrix but a
diagonal matrix may or may not be a scalar matrix.
7)Unit or Identity Matrix: A diagonal matrix in
which all diagonal elements are ‘1’ and all non-
diagonal elements are zero, is called an identity
matrix. It is denoted by I.
1 0 0
e.g. I= [0 1 0 ]
0 0 1

In general, A = [aij]m×n is an identity matrix, if aij = 1,


when i = j and aij = 0, when i ≠ j.
8)Zero or Null Matrix: A matrix is said to be a zero
or null matrix, if its all elements are zero
0 0
e.g. A= [ ]
0 0
9) Upper Triangular Matrix
A square matrix in which all the elements below the
diagonal are zero is known as the upper triangular
matrix. For example,

146
3 −5 7
A= [0 4 0]
0 0 9
10) Lower Triangular Matrix
A square matrix in which all the elements above the
diagonal are zero is known as the upper triangular
matrix. For example,
3 0 0
A= [0 4 0]
7 −5 9

Equality of Matrices: Two matrices A and B are


said to be equal, if
(i) order of A and B are same.
(ii) corresponding elements of A and B are same
i.e. aij = bij, ∀ i and j.

147
148
Operations on Matrices
Between two or more than two matrices, the
following operations are defined below:

Addition and Subtraction of Matrices: Addition


and subtraction of two matrices are defined in an
order of both the matrices are same.
Matrix addition is the operation of adding two or
matrices by adding the corresponding entry of each
matrix together.

149
The most important rule to know is that when
adding two or more matrices, first make sure the
matrices have the same dimensions. In order words,
you can add a 2 x 3 with a 2 x 3 or a 2 x 2 with a 2 x
2. However, you cannot add a 3 x 2 with a 2 x 3 or a
2 x 2 with a 3 x 3. For example, the addition of two
given matrices with dimension 2 × 2,

Addition of Matrix
If A = [aij]m×n and B = [bij]m×n, then A + B =
[aij +bij]m×n, 1 ≤ i ≤ m, 1 ≤ j ≤ n
Properties of Addition of Matrices
(a) Commutative: If A = [aij] and B = [bij] are
matrices of the same order say m x n

Then , A + B = B + A
(b) Associative for any three matrices:

150
A = [aij], B = [bij], C = [cij] of the same order say m x
n,

A + (B + C) = (A + B) + C.
(c) Existence of additive identity:
Let A = [aij] be a mxn matrix and O be a mxn zero
matrix,

Then , A + O = O + A = A.
In other words, O is the additive identity for matrix
addition.
(d) Existence of additive inverse:
Let A = [aij]m×n be any matrix, then we have another
matrix as -A = [-aij]m×n such that

A + (-A) = (-A + A) = O.
So, matrix (-A) is called additive inverse of A or
negative of A.
Note
(i) If A and B are not of the same order, then A + B
is not defined.
(ii) Addition of matrices is an example of a binary
operation on the set of matrices of the same order.
Subtraction of Matrix
If A and B are two matrices of the same order, then
we define A−B=A+(−B).
Consider the two matrices A & B of order 2 x 2.
Then the difference is given by:

151
If A = [aij]m×n and B = [bij]m×n, then A – B =
[aij – bij]m×n, 1 ≤ i ≤ m, 1 ≤ j ≤ n
Properties of Matrix Subtraction
All constraints for the addition of matrices are
applied to the subtraction of matrices as well. But
there are certain laws that matrix subtraction does
not follow just like the subtraction of numbers. The
most important necessity for the subtraction of
matrices to hold all these properties is that the
matrix subtraction is defined only if the order of the
matrices is the same.
• The number of rows and columns should be
the same for the matrix subtraction.
• The subtraction of matrices is not commutative,
that is, A - B ≠ B - A
• The subtraction of matrices is not associative,
that is, (A - B) - C ≠ A - (B - C)

152
• The subtraction of a matrix from itself results in
a null matrix, that is, A - A = O.
• Subtraction of matrices is the addition of the
negative of a matrix to another matrix, that is, A
- B = A + (-B).
Important Notes on Subtraction of Matrices
• Subtraction of matrices is possible only if the
matrices have the same dimension.
• The subtraction of matrices is not commutative
and associative.
• We subtract the corresponding elements of the
matrices for the matrix subtraction.
Example: Write the elements of the matrix C = A -
B explicitly if A = [2 5 9] and B = [1 9 12] using
matrix subtraction formula.
Solution: Since the dimensions of the matrices A
and B are the same, that is, 1 × 3, subtraction of
matrices is possible for these two matrices.
C = A - B = [2-1 5-9 9-12] = [1 -4 -3]
Answer: The elements of C = A - B are c11 = 1, c12
= -4, c13 = -3

153
Multiplication of a matrix by scalar number: Let
A = [aij]m×n be a matrix and k is scalar, then kA is
another matrix obtained by multiplying each
element of A by the scalar k, i.e. if A = [aij]m×n, then
kA = [kaij]m×n.

154
Properties of Scalar Multiplication of a Matrix :
Let A = [aij] and B = [bij]be two matrices of the same
order say m × n, then
(a) k(A + B) = kA + kB, where k is a scalar.
(b) (k + l)A = kA + lA, where k and l are scalars.
Multiplication of Matrices: Let A and B be two
matrices. Then, their product AB is defined, if the
number of columns in matrix A is equal to the
number of rows in matrix B.

Properties of Multiplication of Matrices


(a) Non-commutativity Matrix multiplication is not
commutative i.e. if AB and BA are both defined,
then it is not necessary that AB ≠ BA.
(b) Associative law For three matrices A, B, and C,
if multiplication is defined, then A (BC) = (AB) C.
(c) Multiplicative identity For every square matrix A,
there exists an identity matrix of the same order
such that IA = AI = A.
Note: For Amxm, there is only one multiplicative
identity Im.
(d) Distributive law For three matrices A, B, and C,
A(B + C) = AB + AC
(A + B)C = AC + BC
whenever both sides of the equality are defined.

155
Note: If A and B are two non-zero matrices, then
their product may be a zero matrix.
0 −1
e.g. Suppose A = [ ] and B = [3 5] , then AB
0 2 0 0
0 0
=[ ]
0 0

Two matrices A and B are said to be compatible if


the number of columns in A is equal to the number
of rows in B. That means if A is a matrix of order
m×n and B is a matrix of order n×p, then we can
say that matrices A and B are compatible.

Rules for Matrix Multiplication


As we studied, two matrices can be multiplied only
when they are compatible, which means for the
multiplication of matrices to exist the number of
columns in the first matrix should be equal to the
number of rows in the second matrix, in the above
case 'n'. If A is a matrix of order m×n and B is a
matrix of order n×p, then the order of the product
matrix is m×p.

156
For example,
a) Multiplying a 4 × 3 matrix by a 3 × 4 matrix is
valid and it gives a matrix of order 4 × 4
b) 7 × 1 matrix and 1 × 2 matrices are compatible;
the product gives a 7 × 2 matrix.
c) Multiplication of a 4 × 3 matrix and 2 × 3 matrix is
NOT possible.
Matrix Multiplication Formula
We can understand the general process of matrix
multiplication by the technique, "First rows are
multiplied by columns (element by element) and
then the rows are filled up." Consider two matrices
of order 3×3 as given below,
𝑎 𝑏 𝑐 𝑗 𝑘 𝑙
[𝑑 𝑒 𝑓] & [𝑚 𝑛 𝑜]
𝑔 ℎ 𝑖 𝑝 𝑞 𝑟

157
Here, the matrices have the same dimensions, so
the resultant matrix will also have the same
dimension 3×3

Properties of Matrix Multiplication


There are certain properties of matrix
multiplication operation in linear algebra in
mathematics. These properties are as given below,
• Non-Commutative: Matrix multiplication is
non-commutative, i.e., for multiplication of two
matrices A and B, AB ≠ BA.
• Distributive: Matrix multiplication follows the
distributive property, i.e., multiplication of matrix
A and matrix B with another matrix C, A(B + C)
= AB + BC, given that A, B, and c are
compatible.
• Product with Scalar: If the product of matrices
A and B, AB is defined then, c(AB) = (cA)B =
A(Bc), such that c is a scalar.
• Transpose: The transpose of the product of
matrices A and B can be given as, (AB)T =
BTAT, where T denotes the transpose.
• Complex Conjugate: If A and B are complex
entries, then (AB)* = B*A*

158
• Associativity: Given three matrices A, B and
C, such that the products (AB)C and A(BC) are
defined, then (AB)C = A(BC).
How to Multiply Matrices?
Multiplication of two compatible matrices can be
performed using some general steps as explained
above. The steps in matrix multiplication are
given as,
• Make sure that the number of columns in the
1st matrix equals the number of rows in the 2nd
matrix (compatibility of matrices).
• Multiply the elements of each row of the first
matrix by the elements of each column in the
second matrix.
• Add the products.
• Place the added products in the respective
columns.
Let us understand these steps for multiplication of
matrices better using an example.
Example: Multiply the matrices given below, to
find their product of
1 2 2
[ 3 4] and [ ]
4
5 1
Solution: The given matrices are of order 3×2 and
2×1. ∵The given matrices are compatible, we can
perform the matrix multiplication and the product
matrix will be of order 3×1.

159
1 2
= [3 4] × [2]
4
5 1
(1 × 2) + (2 × 4) 2+8

= [ (3 × 2) + (4 × 4) ] = [ 6 + 16]
(5 × 2 ) + (1 × 4) 10 + 4

10
Answer: Product matrix is [ 22]
14
Important Notes on Multiplication of Matrices:
• To multiply matrices, the given matrices should
be compatible.
• The order of a product matrix can be obtained
by the following rule:
If A is a matrix of order m×n and B is a matrix
of order n×p, then the order of the product
matrix is m×p.
• Matrix multiplication indicates rows by columns
multiplication.
Example 1: Using the matrix multiplication
formula, find the product of the matrices:
1 0
[ ] and [ 6 8]
2 4 4 3

Solution:
The given matrices are of order 2×2. ∵They are
compatible, we can find the multiplication of the
matrices and their product matrix will also be 2×2.
160
1 0 6 8
Product of matrices [ ] and ]is:
[
2 4 4 3
(1 × 6) + (0 × 4) (1 × 8) + (0 × 3)
=[ ]
(2 × 6) + (4 × 4) (2 × 8) + (4 × 3)
6+0 8+0
= [ ]
12 + 16 16 + 12
6 8
=[ ]
28 28
6 8
Answer: Product matrix is [ ]
28 28

Determinant of a Matrix
The determinant of a matrix is the scalar value
computed for a given square matrix. Linear algebra
deals with the determinant, it is computed using the
elements of a square matrix. It can be considered
as the scaling factor for the transformation of a
matrix. Useful in solving a system of linear equation,
calculating the inverse of a matrix and calculus
operations.
Geometrically, the determinant is seen as the
volume scaling factor of the linear transformation
defined by the matrix. It is also expressed as the
volume of the n-dimensional paralleled piped
crossed by the column or row vectors of the matrix.
The determinant is positive or negative as per the
linear mapping preserves or changes the orientation
of n-space.

161
Definition of Determinant of Matrix
The determinant of a matrix is the scalar value or
number calculated using a square matrix. The
square matrix could be 2×2, 3×3, 4×4, or any type,
such as n × n, where the number of column and
rows are equal. If S is the set of square matrices, R
is the set of numbers (real or complex) and f : S →
R is defined by f (A) = k, where A ∈ S and k ∈ R,
then f (A) is called the determinant of A.
To every square matrix A = [aij] of order n, we can
associate a number (real or complex) called
determinant of the square matrix A, where a = (i, j)th
element of A. This may be thought of as a function
which associates each square matrix with a unique
number (real or complex).
If M is the set of square matrices, K is the set of
numbers (real or complex) and f : M → K is defined
by f (A) = k, where A ∈ M and k ∈ K, then f (A) is
called the determinant of A. It is also denoted by | A
| or det A or Δ.
𝑎 𝑏
If A=[ ] , then determinant of A is written as
𝑐 𝑑
𝑎 𝑏
|A| =| |= detA
𝑐 𝑑
Symbol
The determinant of a matrix is represented by two
vertical lines or simply by writing det and writing the
matrix name. eg. |A|, det(A), det A
For a 1×1 Matrix

162
Let A = [a] be the matrix of order 1, then
determinant of A is defined to be equal to a.
For a 2×2 Matrix
For a 2×2 matrix (2 rows and 2 columns):

The determinant is:


|A| = ad − bc or the determinant of A equals
a × d minus b × c.
It is easy to remember when you think of a cross,
where blue is positive that goes diagonally from left
to right and red is negative that goes diagonally
from right to left.

Exa
mple:
2 3
If A= [ ]
4 8

163
|A| = 2 x 8 – 4 x 3
= 16 – 12
=4
For a 3×3 Matrix
For a 3×3 matrix (3 rows and 3 columns):

The determinant is:


|A| = a (ei − fh) − b (di − fg) + c (dh − eg). The
determinant of A equals ‘a times e x i minus f x
h minus b times d x i minus f x g plus c times d
x h minus e x g’.

To work out the determinant of a matrix 3×3:


• Multiply ‘a’ by the determinant of the 2×2 matrix
that is not in a’s row or column.
• Likewise for ‘b’ and for ‘c’

164
• Sum them up, but remember the minus in front
of the b
As a formula (remember the vertical bars || mean
“determinant of”):

The determinant of A equals ‘a’ times the


determinant of e × i minus f × h minus ‘b’ times the
determinant of d × i minus f × g plus ‘c’ times the
determinant of d × h minus e × g.
Example:
6 1 1
If A= [4 −2 5]
2 8 7
|A|= 6×(−2×7 − 5×8) − 1×(4×7 − 5×2) + 1×(4×8 −
(−2×2))
= 6×(−54) − 1×(18) + 1×(36)
= −306

DETERMINANTS

165
Properties of Determinants
Property 1:The value of the determinant remains
unchanged if both rows and columns are
interchanged.
Property 2:If any two rows (or columns) of a
determinant are interchanged, then the sign of
determinant changes.

166
Property 3:If any two rows (or columns) of a
determinant are identical (all corresponding
elements are same), then the value of the
determinant is zero.
Property 4:If each element of a row (or a column)
of a determinant is multiplied by a constant k,
then its value gets multiplied by k.
Property 5:If some or all elements of a row or
column of a determinant are expressed as the
sum of two (or more) terms, then the
determinant can be expressed as the sum of
two (or more) determinants. For example,

Property 6: If the equi-multiples of


corresponding elements of other rows (or
columns) are added to every element of any row
or column of a determinant, then the value of
determinant remains the same, i.e., the value of
determinant remain same if we apply the
operation Ri → Ri + k Rj or Ci → Ci + k Cj .
Minor of a Determinant
A minor is defined as a value computed from the
determinant of a square matrix which is obtained
after crossing out a row and a column
corresponding to the element that is under
consideration. Minor of an element aij of a
determinant is the determinant obtained by deleting
its ith row and jth column in which element aij lies.
Minor of an element aij is denoted by Mij.

167
Co-factor of a Determinant
The co-factor is defined as the signed minor. Co-
factor of an element aij, denoted by Aij is defined by
A = (–1)i+j M, where M is minor of aij.
Note
• We note that if the sum i+j is even, then Aij = Mij,
and that if the sum is odd, then Aij = −Mij.
• Hence, the only difference between the related
minor entries and co-factors may be a sign
change or nothing at all.
• Whether or Aij = Mij or Aij = −Mij has a pattern
for square matrices as illustrated:

For example C12 = −M12. Of course, if you forget,


you can always use the formula Cij = (−1)i+j Mij,
Here, C12=(−1)1+2 Mij = (−1)3 Mij = −Mij
Example
Find the minors and co-factors of all the elements of
the determinant

168
1 −2
| |
4 3
Solution: Minor of the element aij is Mij.
Here a11 = 1. So M11 = Minor of a11 = 3
M12 = Minor of the element a12 = 4
M21 = Minor of the element a21 = –2
M22 = Minor of the element a22 = 1
Now, co-factor of aij is Aij. So,
A11 = (–1)1+1, M11 = (–1)2 (3) = 3
A12 = (–1)1+2, M12 = (–1)3 (4) = –4
A21 = (–1)2+1, M21 = (–1)3 (–2) = 2
A22 = (–1)2+2, M22 = (–1)4 (1) = 1
Definition of Ad-joint of a Matrix
The ad-joint of a square matrix A = [aij]n x n is defined
as the transpose of the matrix [Aij]n x n, where Aij is
the co-factor of the element aij. Ad-joint of the matrix
A is denoted by adj A.

Example :

169
Find the ad-joint of the matrix:

Solution: We will first evaluate the co-factor of every


element,

Therefore,

170
The Relation between Ad-joint and Inverse of a
Matrix
To find the inverse of a matrix A, i.e A-1 we shall first
define the ad-joint of a matrix. Let A be an n x n
matrix. The (i,j) co-factor of A is defined to be
Aij = (-1)ij det(Mij),
where Mij is the (i,j)th minor matrix obtained from A
after removing the ith row and jth column. Let’s
consider the n x n matrix A = (Aij) and define the n x
n matrix Adj(A) = AT. The matrix Adj(A) is called the
ad-joint of matrix A. When A is invertible, then its
inverse can be obtained by the formula given below.

The inverse is defined only for non-singular square


matrices. The following relationship holds between
a matrix and its inverse:

AA-1 = A-1A = I, where I is the identity matrix.


Example :
Determine whether the matrix given below is
invertible and if so, then find the invertible matrix
using the above formula.

171
1 5 2
[0 −1 2 ]
0 0 1
Solution: Since A is an upper triangular matrix, the
determinant of A is the product of its diagonal
entries. This, we have det(A) = -1, which is a non-
zero value and hence, A is invertible. To find the
inverse using the formula, we will first determine the
co-factors Aij of A. We have,

Then the ad-joint matrix of A is

−1 0 0 𝑇 −1 −5 12
[−5 1 0 ] = [0 1 −2 ]
12 −2 −1 0 0 −1

|𝑨|= 1(-1 x 1 - 2 x 0) - 5 (0 x 1 - 0 x 2) +2 (0 x 0 - 0 x -1)


=1( -1 - 0) -5 (0 - 0) + 2 (0 - 0)

=1(-1) - 0 + 0 = -1

172
Using the formula, we will obtain the inverse matrix
as

−𝟏 −𝟓 𝟏𝟐
𝟏
A-1 = −𝟏 × [𝟎 𝟏 −𝟐 ]
𝟎 𝟎 −𝟏
𝟏 𝟓 −𝟏𝟐
= [𝟎 −𝟏 𝟐 ]
𝟎 𝟎 𝟏

Determinants and Matrices as Equation Solver


Here, we will discuss the way to solve a system of
linear equations in two or three variables. With the
help of the determinant, we can also check for the
consistency of linear equations.
• Consistent System: If one or more
solution(s) exists for a system of equations
then it is a consistent system
• Inconsistent System: A system of equations
with no solution is an inconsistent system.

173
The Solution of System of Linear Equations
A solution for a system of linear Equations can be
found by using the inverse of a matrix. Suppose we
have the following system of equations
• a11 x + a12 y + a13 z = b1
• a21 x + a22 y + a23 z = b2
• a31 x + a32 y + a33 z = b3
where, x, y, and z are the variables and a11, a12, … ,
a33 are the respective coefficients of the variables
and b1, b2, and b3 are the constants. We need to
find the solution for the values of the variables in
this system of equations.

Determinant as an Equation Solver


The above system of equations can be represented
in the form of a square matrix as

i.e., AX = B or,

174
Here arise two cases
Case1
If A is a non-singular matrix i.e., |A| ≠ 0, then its
inverse exists.
We have A X = B
or, A– 1 (A X) = A– 1 B (pre-multiplying by A– 1)
or, (A– 1 A) X = A– 1 B
and, I X = A– 1 B (I is the identity matrix)
or, X = A– 1 B where, A– 1 = (adj A) ⁄ |A|
This matrix equation provides a unique solution and
is known as the Matrix Method.

Case2
If A is a singular matrix, then |A| = 0 then we
calculate (adj A) B. If (adj A) B ≠ 0 (zero matrix),
then the solution does not exist. The system of
equations is inconsistent. Else, if (adj A) B = 0 then
the system will either have infinitely many solutions
(consistent system) or no solution (inconsistent
system).

175
Example : Suppose you have three numbers.
The sum of the two numbers and the twice of
the second equals 2. The sum of the second and
third when subtracted from the twice of first
gives 1. The difference of thrice of first and five
times the third gives 5. Rewrite the statement in
form of the system of equations. Solve it using
Matrix Method as an equation solver.
Answer : Assume that x, y, and z are the three
numbers. Rewriting the above statement we have
the following system of equations

x + 2y + z = 2

2x – y – z = 1

3x – 5y = 5

In matrix notation, we have

Here, the determinant of A =

176
|A| = 1(5 – 0) – 2(–10 + 3) + 1(0 + 3)
= 22 ≠ 0.
Hence there exists a unique solution for X.
Calculating adj (A), we have Aij = (–1)(i + j) Mij , where
Mij is the co-factor of aij
• A11 = 1(5 – 0) = 5, A12 = –1(–10 + 3) = 7, A13 =
1(0 + 3) = 3,
• A21 = –1(–10 –0) = 10, A22 = 1(–5 – 3) = –8, A23

= –1(0 – 6) = 6,
• A31 = 1(–2 + 1) = –1, A32 = –1(–1 – 2) = 3, A33 =

1(–1 – 4) = –5

The inverse of the matrix A is A−1.

177
Since X = A– 1 B

Thus, x = 15⁄22,

y = 21⁄22,
z = –13⁄22.

178
System of Linear equations:
(i) Matrix Inversion Method:
This method can be applied only when the coefficient
matrix is a square matrix and non- singular.

Consider the matrix equation

AX = B , … (1)

where A is a square matrix and non-singular. Since


A is non-singular, A−1 exists and A−1 A =
AA−1 = I.

Pre-multiplying both sides of (1) by A−1, we get A−1


( AX ) = A−1B. That is,

( A−1 A) X = A−1B.

Hence, we get

X = A−1B.

179
Example1:Solve the following system of linear
equations, using matrix inversion method:

5x + 2 y = 3,
3x + 2 y = 5 .
Solution
The matrix form of the system is AX = B ,

where
5 2
We find |A| = | |= 10 - 6= 4 ≠ 0.
3 2
So, A−1 exists and
1 2 −2
A−1 = [ ]
4 −3 5

Then, applying the formula X = A−1B , we get

So the solution is (x = −1, y = 4).


Example 2:

180
Solve the following system of equations, using
matrix inversion method:
2x1 + 3x2 + 3x3 = 5,
x1 – 2x2 + x3 = -4,
3x1 – x2 – 2x3 = 3

Solution
The matrix form of the system is AX = B,where

So, the solution is ( x1 = 1, x2 = 2, x3 = −1) .

181
Example 3:
If

,
find the products AB and BA and hence solve the
system of equations x − y + z = 4, x – 2y – 2z = 9, 2x
+ y +3z =1.
Solution

Writing the given system of equations in matrix form,


we get

182
Hence, the solution is (x = 3, y = - 2, z = −1).

(ii) Cramer’s Rule


This rule can be applied only when the coefficient
matrix is a square matrix and non-singular. It is
explained by considering the following system of
equations:

183
𝑎11 𝑎12 𝑎13
where the coefficient matrix [𝑎21 𝑎22 𝑎23 ] is
𝑎31 𝑎32 𝑎33
𝑎11 𝑎12 𝑎13
non-singular. Then |𝑎21 𝑎22 𝑎23 |≠ 0
𝑎31 𝑎32 𝑎33

𝑎11 𝑎12 𝑎13


Let us put Δ = |𝑎21 𝑎22 𝑎23 |. Then, we have
𝑎31 𝑎32 𝑎33

Note

184
Replacing the first column elements a11 , a21 , a31 of Δ
with b1 , b2 , b3 respectively, we get Δ1. Replacing the
second column elements a12 , a22 , a32 of Δ with b1 ,
b2 , b3 respectively, we get Δ2 . Replacing the third
column elements a13 , a23 , a33 of Δ with b1 , b2 ,b3
respectively, we get Δ3.

If Δ = 0, Cramer’s rule cannot be applied.

Example 1:

Solve, by Cramer’s rule, the system of equations

x1 − x2 = 3,
2x1 + 3x2 + 4x3 = 17,
x2 + 2x3 = 7.
Solution
First we evaluate the determinants

185
So, the solution is (x1 = 2, x2 = - 1, x3 = 4).

Example 2:
In a T20 match, Chennai Super Kings needed just 6
runs to win with 1 ball left to go in the last over. The
last ball was bowled and the batsman at the crease
hit it high up. The ball traversed along a path in a
vertical plane and the equation of the path
is y = ax2 + bx + c with respect to a xy -coordinate
system in
the vertical plane and the ball traversed throu
gh the points (10,8), (20,16), (30,18) , can you
conclude that Chennai Super Kings won the match?
Justify your answer. (All distances are measured in
metres and the meeting point of the plane of the path
with the farthest boundary line is (70, 0).)

186
Solution
The path y = ax2 + bx + c passes through the
points (10,8), (20,16), (40, 22) . So, we
get the system of equations
100a + 10b + c = 8,
400a + 20b + c= 16,
1600a + 40b + c = 22.
To apply Cramer’s rule, we find

187
When x = 70, we get y = 6.
So, the ball went by 6 meters high over the boundary
line and it is impossible for afielder standing
even just before the boundary line to jump and catch
the ball.
Hence the ball went for a super six and the Chennai
Super Kings won the match.

(iii) Gaussian Elimination Method


This method can be applied even if the coefficient
matrix is singular matrix and rectangular matrix. It is
essentially the method of substitution which we
have already seen. In this method, we transform the
augmented matrix of the system of linear equations
into row-echelon form and then by back-substitution,
we get the solution.
Example 1:
Solve the following system of linear equations, by
Gaussian elimination method :
4x + 3y + 6z = 25,
x + 5 y + 7z = 13,
2x + 9 y + z = 1.

Solution

188
Transforming the augmented matrix to echelon form,
we get

R2 → R2 x (-1)
R3 →R3 x( -1)

The equivalent system is written by using the


echelon form:
x + 5y + 7z = 13 , … (1)
17y + 22z = 27 , … (2)
199z = 398 . … (3)

189
Substituting z = 2, y = -1 in (1), we get x = 13 - 5 ×
(−1 ) − 7 × 2 = 4 .
So, the solution is ( x =4, y = - 1, z = 2 ).
Note. The above method of going from the last
equation to the first equation is called the method
of back substitution.

Example 2:
The upward speed v(t) of a rocket at time t is
approximated by v(t) = at2 + bt + c, 0 ≤ t ≤ 100 where
a, b, and c are constants. It has been found that the
speed at times t = 3, t = 6 , and t = 9 seconds are
respectively, 64, 133, and 208 miles per second
respectively. Find the speed at time t = 15 seconds.
(Use Gaussian elimination method.)

Solution
Since v(3) =64, v(6) = 133 and v(9) = 208 , we get
the following system of linear equations
9a +3b + c = 64 ,
36a + 6b + c = 133,
81a + 9b + c = 208 .
We solve the above system of linear equations by
Gaussian elimination method.

190
Reducing the augmented matrix to an equivalent
row-echelon form by using elementary row
operations, we get

Writing the equivalent equations from the row-


echelon matrix, we get
9a + 3b + c = 64, 2b + c = 41, c= 1.
By back substitution, we get

So, we get v (t) = 1/3 t2 + 20t + 1.


Hence, v(15) = 1/3 (225) + 20(15) + 1 = 75 + 300 +
1 = 376

191
UNIT 5:
TIME VALUE OF MONEY AND SIMPLE
ANNUITY
(Introduction of Annuity, Types
of Annuity, Amount and
Present Value of Immediate
(NPV), Annuity, Annuity Due)

192
What is an Annuity?
Many people have had the experience of making a series
of fixed payments over a course of time - such as rent,
premium or vehicle payments - or obtaining a series of
payments for a course of time, such as the certificate of
deposit (CD) or interest from a bond or lending money.
These ongoing or recurring payments are technically called
"annuities.”
An annuity is a financial product that provides certain
cash flows at equal time intervals. Annuities are
created by financial institutions, primarily life
insurance companies, to provide regular income to
a client.
An annuity is a series of payments made at equal
intervals. Examples of annuities are regular deposits
to a savings account, monthly home mortgage
payments, monthly insurance payments and
pension payments. Annuities can be classified by
the frequency of payment dates. The payments
(deposits) may be made weekly, monthly, quarterly,
yearly, or at any other regular interval of time.

Annuities may be calculated by mathematical


functions known as "annuity functions".
Annuities are insurance contracts that provide a
fixed income stream for a person’s lifetime or a
specified period. An annuity can be purchased with
a lump sum or a series of paymentsand begin paying
out almost immediately or at some point in the
future. Annuities are often used as a way to fund
retirement.
An annuity is a reasonable alternative to some
193
other investments as a source of income since it
provides guaranteed income to an individual.

194
Additional features of Annuities
However, annuities are less liquid than investments
in securities because the initially deposited lump
sum cannot be withdrawn without penalties.
Upon the issuance of an annuity, an individual pays
a lump sum to the issuer of the annuity (financial
institution). Then, the issuer holds the amount for a
certain period (called an accumulation period). After
the accumulation period, the issuer must make fixed
payments to the individual according to
predetermined time intervals.
Annuities are primarily bought by individuals who
want to receive stable retirement income.
An annuity is a customizable contract issued by an
insurance company that converts an investor’s
premiums into a guaranteed fixed income stream.
The type of annuity you purchase determines your
future annuity payments.
The primary benefits of buying an annuity include
principal protection, the potential for guaranteed
lifetime income and the option to leave money to
your beneficiaries. Some annuities may also be
optimized to help pay for long-term care.
An annuity formula is used to find the present and
future value of an amount. An annuity is a fixed
amount of income that is given annually or at
regular intervals. An annuity is an agreement with
an insurance company in which you make a lump
sum payment (one-time big payment) or series of
payments and, in return, receive a regular fixed

195
income, beginning either immediately or after some
predefined time in the future.
Types of Annuities
There are several types of annuities that are
classified according to frequency and types of
payments. For example, the cash flows of annuities
can be paid at different time intervals. The payments
can be made weekly, biweekly, or monthly. The
primary types of annuities are:
1. Fixed annuities: Annuities that provide fixed
payments. The payments are guaranteed, but the
rate of return is usually minimal.
2. Variable annuities: Annuities that allow an
individual to choose a selection of investments that
will pay an income based on the performance of the
selected investments. Variable annuities do not
guarantee the amount of income, but the rate of
return is generally higher relative to fixed annuities.
3. Life annuities: Life annuities provide fixed
payments to their holders until his/her death.
4. Perpetuity: An annuity that provides perpetual
cash flows with no end date. Examples of financial
instruments that grant perpetual cash flows to its
holder are extremely rare.
Two Types of Annuities
Annuities, in this sense of the word, break down
into two basic types: ordinary annuities and
annuities due.

196
• Ordinary annuities: An ordinary annuity
makes (or requires) payments at the end of
each period. For example, bonds generally
pay interest at the end of every six months.
• Annuities due: With an annuity due, by
contrast, payments come at the beginning
of each period. Rent, which landlords
typically require at the beginning of each
month, is a common example.

Present Value of an Annuity


The present value of an annuity is based on the
concept of the time value of money. The time
value of money means that money is worth more the
sooner you have it. Payments that are scheduled to
be received in the future are worth less today
because of the uncertainty of future economic
conditions. Current payments have more value
because they can be invested in the meantime.

197
The present value of an annuity is the current value
of the future payments that the annuitant will
receive, given a specified rate of return. The present
value measures the current cash value of all of the
future payments that the annuitant will receive.
The present value calculation is useful in
determining whether the annuitant will receive more
money by taking an immediate lump sum or
spreading out annuity payments over a longer period
of time.

198
Future Value of an Annuity
The present value of an annuity is based on the
concept of the time value of money. The time value
of money means that money is worth more the
sooner you have it. Payments that are scheduled to
be received in the future are worth less today
because of the uncertainty of future economic
conditions. Current payments have more value
because they can be invested in the meantime.
The future value of an annuity is the total value that
annuity payments will be worth at a specific point in
the future. It is the value of a group of recurring
payments at a specific date in the future, given a
particular rate of return. The higher the rate of

199
return is, the greater the annuity’s future value will
be.
The future value of an annuity differs from the
present value of an annuity, as the present value
gives you the current value of future annuity
payments.
Knowing the future value of your annuity is useful for
annuitants who want to plan for retirement. By
knowing how much annuity payments will be worth,
annuitants can accurately plan how to allocate other
sources of income and how to handle other
investments.

200
How to Calculate Annuities
There are various ways to measure the annuity rate
changes or the cost of making such payments or what
they're ultimately worth. However, it is first better to know
about calculating the present value of the annuity or the
future value of the annuity.

Formula to Calculate Present Value Annuities


The formula for the present value of an ordinary
annuity:
−𝐧
PV ordinary annuity = P × 𝟏 − (𝟏 + 𝐫)
𝐫
Where,
PV = present value of an ordinary annuity
P = value of each payment

201
R = interest rate/ period
N = total number of periods
The formula for calculating the present value of an
annuity due is:
PV Annuity Due = C × [i1 − (1 + i)−n] × (1 + i)

Formula to Calculate Future Value Annuities


Instead of calculating each payment separately and then
adding them all up, you can instead apply the following
formula, which will tell you the amount of money you'd have
in the end:
FV Ordinary Annuity = C × [i(1 + i)n −1]
Where:
C = cash flow/period
i = rate of interest
n = total number of payments
The formula for the future value of an annuity due is:
FV Annuity Due = C × [i(1 + i)n−1] × (1 + i)

Solved Examples
Example:
Calculate the future value of the ordinary annuity and the
present value of an annuity due where cash flow per period
amounts to rs. 1000 and interest rate is charged at 0.05%.

202
Solution:
Using the formula to calculate future value of ordinary
annuity = C × [(1 + i)n – 1/i
= Rs. 1,000 × [0.05 (1 + 0.05)5−1]
=Rs.1, 000 × 5.53
=Rs. 5,525.63
Note that the one-cent difference in these outcomes, Rs.
5,525.64 vs. Rs. 5,525.63, is because of rounding in the first
calculation.
Now to calculate the present value of an annuity due:
Use the formula
PV Annuity Due = C × [i1 − (1 + i)−n] × (1 + i)
Plugging in the values:
= Rs. 1,000 × [0.05(1− (1 + 0.05)−5] × (1 + 0.05)
= Rs. 1,000 × 4.33 × 1.05
= Rs. 4,545.95

Did You Know?


⚫ Annuities are applicable when you are saving money.
⚫ Generally in an annuity problem, your account begins
empty but has money in the future.
⚫ Annuities suppose that you put money in the account
on a routine basis (every month, quarter year, etc.) and
let it remain to earn interest.
⚫ If you’re putting money into the account on a regular
basis, then you’re looking at a basic annuity problem.
⚫ Recurring payments, such as the rent or interest are
sometimes referred to as "annuities".

203
⚫ The present value of the annuity is the amount of money
that would be needed now to generate those future
payments.
⚫ The future value of the annuity is the total value of
payments at a particular point in time.
⚫ In ordinary annuities, payments are released at the end
of each time period.
⚫ With annuities due, they're made at the commencement
of the period.

Valuation of Annuities:
Annuities are valued by discounting the future cash
flows of the annuities and finding the present value
of the cash flows. The general formula for annuity
valuation is:

Where:
• PV = Present value of the annuity
• P = Fixed payment
• r = Interest rate
• n = Total number of periods of annuity
payments

204
The valuation of perpetuity is different because it
does not include a specified end date. Therefore,
the value of the perpetuity is found using the
following formula:
PV = P / r

What is Annuity Formula?


The annuity formula helps in determining the values
for annuity payment and annuity due based on the
present value of an annuity due, effective interest
rate, and several periods. Hence, the formula is
based on an ordinary annuity that is calculated
based on the present value of an ordinary annuity,
effective interest rate, and several periods. The
annuity formulas are:
Annuity = r * PVA Ordinary / [1 – (1 + r)-n]
Annuity = r * PVA Due / [{1 – (1 + r)-n} * (1 + r)]
. The Annuity Formulas for future value and present
value are:
The future value of an annuity,
FV = P×((1+r)n−1) / r
The present value of an annuity,
PV = P×(1−(1+r)-n) / r

205
Annuity Formula
The formula is calculated based on two important
aspects - The present Value of the Ordinary Annuity
and the Present Value of the Due Annuity.
Annuity = r * PVA Ordinary / [1 – (1 + r)-n]
Where,
• PVA Ordinary = Present value of an ordinary
annuity
• r = Effective interest rate
• n = Number of periods
Annuity = r * PVA Due / [{1 – (1 + r)-n} * (1 + r)]
Where,

206
• PVA Due = Present value of an annuity due
• r = Effective interest rate
• n = number of periods
The Annuity Formulas for future value and present
value is:
The future value of an annuity,
(𝟏+𝐫)𝐧−𝟏)
FV = P×( )
𝐫

The present value of an annuity,


𝟏− (𝟏+𝐫)−𝐧)
PV = P×( )
𝐫
where,
• P = Value of each payment
• r = Rate of interest per period in decimal
• n = Number of periods

Practice Questions :
Example 1: Dan was getting Rs.100 for 5 years every
year at an interest rate of 5%. Find the future value
of this annuity at the end of 5 years? Calculate it by
using the annuity formula.
Solution
The future value
Given: r = 0.05, 5 years = 5 yearly payments, so n
= 5, and P = Rs.100

207
(𝟏+𝐫)𝐧−𝟏
FV = P×( )
𝐫
FV = Rs.100 × ((1+0.05)5−1) / 0.05
FV = 100 × 55.256
FV = Rs.5525.6
Therefore, the future value of annuity after the end
of 5 years is Rs.552.56.
Example 2: If the present value of the annuity is
Rs.20,000. Assuming a monthly interest rate of
0.5%, find the value of each payment after every
month for 10 years. Calculate it by using the annuity
formula.
Solution:
Given:
r = 0.5% = 0.005
n = 10 years x 12 months = 120, and
PV = Rs.20,000
Using formula for present value
𝟏−(𝟏+𝐫)−𝐧
PV = P×( )
𝐫

Or, P = PV × ( r / (1−(1+r)−n))
P = Rs.20,000 × (0.005 / (1−(1.005)−120))P
= Rs.20,000 × (0.005/ (1−0.54963))

208
P = Rs.20,000 × 0.011...
P = Rs.220
Therefore, the value of each payment is Rs.220.
Example 3: Jane won a lottery worth Rs.20,000,000
and has opted for an annuity payment at the end of
each year for the next 10 years as a payout option.
Determine the amount that Jane will be paid as
annuity payment if the constant rate of interest in the
market is 5%.
Solution:
Given:
PVA (ordinary) = Rs.20,000,000 (since the annuity
to be paid at the end of each year)
r = 5%
n = 10 years
Using the Annuity Formula,
Annuity = r * PVA Ordinary / [1 – (1 + r)-n]
Annuity = 5% × 20000000 / [1 - (1 + 0.05)-10
Annuity = Rs.2,564,102.56
Therefore, Jane will pay an annuity amount of
Rs.2,564,102.56

209
How to Use the Log Table?
To find the logarithm of a number, we can use the
logarithm table instead of using a mere calculation.
Before finding the logarithm of a number, we should
know about the characteristic part and mantissa part
of a given number
• Characteristic Part – The whole part of a
number is called the characteristic part. The
characteristic of any number greater than one
is positive, and if it is one less than the number
of digits to the left of the decimal point in a given
number. If the number is less than one, the
characteristic is negative and is one more than
the number of zeros to the right of the decimal
point.
• Mantissa Part – The decimal part of the
logarithm number is said to be the mantissa
part and it should always be a positive value. If
the mantissa part is in a negative value, then
convert into the positive value.
The procedure is given below to find the log value
of a number using the log table. First, you have to
know how to use the log table. The log table is
given for the reference to find the values.
Step 1: Understand the concept of the logarithm.
Each log table is only usable with a certain base. The
most common type of logarithm table is used islog
base 10.

210
Step 2: Identify the characteristic part and mantissa
part of the given number. For example, if you want
to find the value of log10 (15.27), first separate the
characteristic part and the mantissa part.
Characteristic Part = 15
Mantissa part = 27
Step 3: Use a common log table. Now, use row
number 15 and check column number 2 and write
the corresponding value. So the value obtained is
1818.
Step 4: Use the logarithm table with a mean
difference. Slide your finger in the mean difference
column number 7 and row number 15, and write
down the corresponding value as 20.

211
Step 5: Add both the values obtained in step 3 and
step 4. That is 1818+20= 1838. Therefore, the value
1838 is the mantissa part.

Step 6: Find the characteristic part. Since the


number lies between 10 and 100, (101 and 102), the
characteristic part should be 1.
Step 7: Finally combine both the characteristic part
and the mantissa part, it becomes 1.1838.

Sample Example

212
Here the sample example to find the value of the
logarithmic function using the logarithm table is
given.
Question:
Find the value of log10 2.872
Solution:
Step 1: Characteristic Part= 2 and mantissa part=
872
Step 2: Check the row number 28 and column
number 7. So the value obtained is 4579.
Step 3: Check the mean difference value for row
number 28 and mean difference column 2. The
value corresponding to the row and column is 3
Step 4: Add the values obtained in step 2 and 3, we
get 4582. This is the mantissa part.
Step 5: Since the number of digits to the left side of
the decimal part is 1, the characteristic part is less
than 1. So the characteristic part is 0
Step 6: Finally combine the characteristic part and
the mantissa part. So it becomes 0.4582.
Therefore, the value of log 2.872 is 0.4582.
How to Calculate Anti-log?
Anti-log Definition: The Anti-log, which is also
known as “Anti- Logarithms” of a number is the
inverse technique of finding the logarithm of the

213
same number. Consider, if x is the logarithm of a
number y with base b, then we can say y is the anti-
log of x to the base b. It is defined by

If logb y = x Then, y = antilog x

Before finding the anti-log of a number, we should


know about the parts like the characteristic and
mantissa part.
• Characteristic Part – The whole part is called
the characteristic part. If the characteristic of
logarithm of any number greater than one is
positive and is one less than the number of
digits in the left side of the decimal point.
• Mantissa Part – The decimal part of the
logarithm number for a given number is called
the mantissa part, and it should always be a
positive value. If the mantissa part is in a
negative value, convert into the positive value.
Procedure to Find the Anti-log of a Number
Method 1: Using an Anti-log Table
Consider a number, 2.6452
Step 1: Separate the characteristic part and the
mantissa part. From the given example, the
characteristic part is 2, and the mantissa part is
6452.

214
Step 2: To find a corresponding value of the mantissa
part use the anti-log table. Using the anti- log table,
find the corresponding value. Now, find the row
number that starts with .64, then the column for 5.
Now, you get the corresponding value as4416.
Step 3: From mean difference columns find the
value. Again use the same row number .64 and find
the value for column 2. Now, the valuecorresponding
to this is 2.
Step 4: Add the values obtained in step 2 and 3, we
get 4416 + 2 = 4418.
Step 5: Now insert the decimal point. The decimal
point always goes the designated place. For this,
you have to add 1 to the characteristic value. Now
you get 3. Then add the decimal point after 3 digits,
we get 441.8
So the anti-log value of 2.6452 is 441.8.
Sample Example
Question:
Find the antilog of 3.3010
Solution:
Given, antilog (3.3010)
Step 1: Characteristics part = 3 and mantissa part =
3010

215
Step 2: Use the antilog table for the row.30, then
the column for 1, you get 2000.
Step 3: Find the value from the mean difference
column for the row .30 and column 0, it gives the
value 0
Step 4: Add the values obtained in step 2 and 3,
2000 + 0 = 2000.
Step 5: Now insert the decimal place. We know that
the characteristic part is 3 and we must add it with 1.
Therefore, we get the value 4. Insert the decimal
point after 4 places, and we get 2000.
Therefore, the solution of the antilog 3.3010 is
2000.

216
217
218
219
220
Previous Year
Question Paper

221
222
223
224
225
226
227

You might also like